PR Outline Final

You might also like

Download as doc, pdf, or txt
Download as doc, pdf, or txt
You are on page 1of 57

MM PROFESSIONAL RESPONSIBILITY PROF. COQUILLETTE SPRING 2009 THE DEAN WANTS ME TO SAY EVERYONE IS SMART AND TALENTED.

THATS A LIE. PROF. COQUILLETTE Previous Exam Hypotheticals................................................................................................................. 2 Regulating the Profession ....................................................................................................................... 6 THE SOURCES OF PROFESSIONAL ETHICS TODAY ...................................................................................................... 6 DEEP THEORIES ................................................................................................................................................................. 7 THE HISTORICAL AND MODERN CONTEXT ................................................................................................................. 7 WHO SHOULD MAKE THE RULES GOVERNING LAWYERS IN FEDERAL MATTERS ............................................... 7 TOWARD AN ECCLESIASTICAL PROFESSIONAL ETHICS .............................................................................................. 8 ABA MODEL RULES: A LAWYERS RESPONSIBILITY ................................................................................................... 8 Loyalty to Present Clients: Concurrent Conflicts of Interest .................................................................. 8 MODEL RULE 1.7 CONFLICT OF INTEREST: CONCURRENT CLIENTS.................................................................... 8 MODEL RULE 1.8 CONFLICTS OF INTERESTS CURRENT CLIENTS SPECIFIC RULES ...................................10 THE LEGAL ETHICS OF LOUIS D. BRANDEIS ..............................................................................................................11 Loyalty to Past Clients: Successive and Imputed Conflicts of Interest .................................................. 11 LEGAL FICTIONS ..............................................................................................................................................................11 MODEL RULE 1.9 DUTIES TO FORMER CLIENTS ....................................................................................................11 MODEL RULE 1.10 IMPUTATION OF CONFLICTS OF INTEREST ............................................................................12 MODEL RULE 1.11 SPECIAL CONFLICTS OF INTEREST FOR FORMER AND CURRENT GOVERNMENT OFFICERS AND EMPLOYEES ...........................................................................................................................................14 TYPES OF INFORMATION ................................................................................................................................................15 PHYSICAL EVIDENCE.......................................................................................................................................................16 MODEL RULE 1.6 CONFIDENTIALITY OF INFORMATION ......................................................................................16 MASS. RULE 1.6. ................................................................................................................................................................18 MODEL RULE 1.6 V. MASS. RULE 1.6 ............................................................................................................................19 MODEL RULE 3.4 FAIRNESS TO OPPOSING PARTY AND COUNSEL .....................................................................19 MODEL RULE 4.4 RESPECT FOR RIGHTS OF THIRD PERSONS ..............................................................................20 Truthfulness ...........................................................................................................................................20 MODEL RULE 3.3 CANDOR TOWARD THE TRIBUNAL ...........................................................................................20 MASS. RULE 3.3(E) CANDOR TOWARD THE TRIBUNAL..........................................................................................22 MODEL RULE 4.1 TRUTHFULNESS IN STATEMENTS TO OTHERS .........................................................................22 MODEL RULE 4.3 DEALING WITH UNREPRESENTED PERSONS..........................................................................24 Communications and Difficult Clients ..................................................................................................24 MODEL RULE 1.4 COMMUNICATIONS .......................................................................................................................24 MODEL RULE 1.13 ORGANIZATION AS CLIENT......................................................................................................24 SARBANES-OXLEY ACT STANDARDS OF PROFESSIONAL CONDUCT FOR ATTORNEYS ...................................26 MODEL RULE 1.14 CLIENT WITH DIMINISHED CAPACITY ...................................................................................27 MODEL RULE 1.16 DECLINING OR TERMINATING REPRESENTATION ..............................................................27 MODEL RULE 2.4 LAWYER SERVING AS THIRD-PARTY NEUTRAL ......................................................................27 MODEL RULE 4.2 COMMUNICATION WITH PERSON REPRESENTED BY COUNSEL ..........................................28 Starting A Law Practice ..........................................................................................................................28 MODEL RULE 1.2 SCOPE OF REPRESENTATION & ALLOCATION OF AUTHORITY ...........................................28 MODEL RULE 1.5 FEES ................................................................................................................................................29 MODEL RULE 5.1 RESPONSIBILITIES OF PARTNERS, MANAGERS & SUPERVISORY LAWYERS .......................30 MODEL RULE 5.2 RESPONSIBILITIES OF A SUBORDINATE LAWYER....................................................................30 MODEL RULE 5.4 PROFESSIONAL INDEPENDENCE OF A LAWYER .....................................................................30 MODEL RULE 7.1 COMMUNICATIONS CONCERNING A LAWYERS SERVICES...................................................31 MODEL RULE 7.2 ADVERTISING ................................................................................................................................31 MODEL RULE 7.3 DIRECT CONTACT WITH PROSPECTIVE CLIENTS (SOLICITATION) .....................................31

MODEL RULE 7.4 COMMUNICATION OF FIELDS OF PRACTICE AND SPECIALIZATION ...................................33 MODEL RULE 7.5 FIRM NAMES AND LETTERHEADS .............................................................................................34 Bar Admissions ......................................................................................................................................34 THE GATEKEEPERS AND YOU .......................................................................................................................................34 MODEL RULE 8.1 BAR ADMISSION AND DISCIPLINARY MATTERS......................................................................34 Bar Discipline and Malpractice .............................................................................................................37 THE DISCIPLINARY PROCESS (IN MASS.)......................................................................................................................37 MODEL RULE 8.3 REPORTING PROFESSIONAL MISCONDUCT (SNITCH RULE).............................................37 MODEL RULE 8.4 MISCONDUCT.................................................................................................................................37 MODEL RULE 8.5 DISCIPLINARY AUTHORITY CHOICE OF LAW.......................................................................40 MAJOR ISSUES IN BAR DISCIPLINE ................................................................................................................................41 Lawyers and Fundamental Moral Responsibility .................................................................................. 41 THE MORAL PERSON .......................................................................................................................................................41 MORAL RESPONSIBILITY .................................................................................................................................................42 THE LEGAL MENTALITY.................................................................................................................................................42 THE MORAL CONSTRAINTS ON RULE MAKING .........................................................................................................44 NATURAL LAW: THE LAW IN THE SKY AND IN THE CAVE .......................................................................................46 JUSTIFIED DISOBEDIENCE..............................................................................................................................................51 THE VALUE OF ALLEGIANCE.........................................................................................................................................56 DUTY FOR DUTYS SAKE .................................................................................................................................................56

I. Previous Exam Hypotheticals Guantanamo Bay Hypo [A-C Privilege, Confidentiality and Multiple State Bars]
Involves Preamble, MR 1.6 and 8.4 Involves Bluebook: Lon Fuller, John Locke. Would I go? I would agree to the terms. Although one might find the modifications of the protections traditionally afforded under the attorney-client privilege, a positivist could simply argue that the existence of law is one thing; its merit or demerit another. Whether a law is valid does not depend on the extent to which it satisfies ideals of justice, democracy, or the rule of law. What laws are in force in that system depends on what social standards its officials recognize as authoritative; for example, legislative enactments, judicial decisions, or social customs. The fact that one might find a policy or law is unjust, unwise, inefficient or imprudent is never sufficient reason for doubting it. In this case, the law was passed by a sovereign, was made public, and is (assumingly) backed up by coercive sanctions against an attorney who violates the rule. On that account, one would either have to comply or choose not to go at all. However, on the other hand, one could make an argument that his or her time would be better spent drawing attention to the denial of privileges that we accord other citizens. Perhaps non-participation will most effectively serve to improve the law and promote the administration of justice. As stated in the MR Preamble, a lawyer should further the publics understanding of and confidence in the rule of law and the justice system and should devote professional time and resources and use civic influence to ensure equal access to our system of justice for all those who cannot afford or secure adequate legal counsel. A lawyer's privilege and duty to press a full and zealous defense on behalf of a client, and the protection of attorney-client confidences, are integral to the independence of the bar and, hence, to our system of justice. One may see any encroachment on these prerogatives as reason enough to refuse to participate in commission proceedings in essence refusing to participate in a modern day star chamber. In this case its is not hard to imagine that the public would lose trust in the judicial system if it sees that the system is changing the rules of law retroactively and selectively characteristics that Lon Fuller articulated as two of the pitfalls in rulemaking. However, if an attorney were to start some form of an awareness campaign informing the public about the denial of certain attorney-client privileges, he or she would have to be careful in what they say or publish. Depending on which vein of caselaw one follows, the courts have held that while the First Amendment protects some free speech, such speech is restricted when it has a reasonable likelihood of interfering with a criminal proceeding and an attorney can be found to have violated MR 8.4(d) engaging in conduct that is prejudicial to the administration of justice. In In Re Hinds, the court considered such factors as the consideration of the status of the attorney, the nature and timing of the statement, as well as the context in which it was uttered. Moreover, while a few mere utterances will not necessarily violate MR 8.4(d), according to the New York Court of Appeals, launching an information campaign that questioned and insulted the

judicial process in Guantanamo Bay might be considered a persistent or general courses of conduct that is degrading to the law, the Bar, and the courts and a violation of MR 8.4(d). Yet, in light of the broader circumstances, it would seem likely that the average reasonable attorney would realize that granting assistance to a defendant already in a perilous situation would be more beneficial than spending his or her time advocating for a change in the law. How do I feel about limiting the A-C Privilege? I do not think that there would be an issue regarding the modification of the attorney-client privilege and an attorneys representation of a Guantanamo Bay defendant. Although the DoDs affidavit states that a civilian defense attorney understands that the traditionally privileged communication will be subject to possible third-party monitoring, MR 1.6(b)(6) states that an attorney may reveal information to comply with other law or a court order. One could argue that submitting to the modified attorney-client privilege is complying with the current law in Guantanamo Bay constitutional issues aside. This, however, raises unique possible problems. First and foremost, since the communications might be monitored, the client may instruct the attorney to refrain from bringing in materials or discussing information that although possibly helpful in his or her defense, might implicate a third-party that the client wishes to remain anonymous. Obviously, this would significantly hamper the attorneys ability to zealously represent his client. It would be reasonable to expect that disclosure to the client regarding the third-parry monitoring would chill the exchange between defense counsel and his client. Moreover, it is questionable whether or not the attorney can actually sign away his clients rights to confidentiality. After all, the privilege is the clients, not the attorneys. Secondly, the attorney may face a serious moral dilemma when deciding whether or not to accept representation in the first place. One could argue that by participating under these restrictive conditions the attorney is condoning the law that exists in Guantanamo Bay that arguably denies defendants their fundamental rights. Member of the Mass. Bar? If during the course of representing the defendant, he or she should file a complaint with the Massachusetts Board of Bar Overseers stating that I violated MR 1.6 by failing to perform my obligation to maintain confidentiality of information obtained during the client-lawyer relationship, I would have a clear response in my defense. I would first point out that I was complying with the law as per the requirements of MR 1.6(b)(6). Whether the law itself is valid, invalid, moral or immoral is not of particular legal concern to interpreting MR 1.6. The Model Rules do not provide a standard for determining the validity of the law but only serves as a guideline for practicing responsibility within the law. Moreover, if I were forced to disclose a piece of information to the Chief Defense Counsel as per the requirements of the DoD affidavit, there still would be no problem as far as violating MR 1.6. Although I might be personally dissatisfied with the DoD taking away my discretion (the may) in reporting information to prevent reasonably certain death or substantial bodily harm, as an attorney I still have the option to exercise that right. If however, the Mass. Bar was to disagree and consider the complaint, it could be problematic. Member of another state bar? If I was a member of another state bar, I might be subject to the same BBO-1 complaint for violating that states version of MR 1.6. As MR 8.5 generally states, a lawyer admitted to practice in a jurisdiction is subject to the disciplinary authority of that jurisdiction, regardless of where the lawyer's conduct occurs. The rule does include some hedging language with regard to the violation occurring during the course of a tribunal, in which case, the local jurisdictional rules apply. However, the rule also includes language to the effect that if the effect of the violation occurs in another jurisdiction, it might violate that jurisdictions rules of professional conduct. Position of the ABA? It is highly likely that the ABA, especially the section of defense lawyers, would object to the modification of the attorney-client privilege. They would likely object on two separate and sometimes conflicting grounds. First, the ABA would probably make some form of argument based on the guild ideal. Attorneys are part of a self-regulating profession that is guided by their own professional values. Although this might sometimes conflict with the interests of the government, attorneys are not supposed to be part of the government apparatus. To allow the modified privilege to stand would discourage individuals from seeking legal counsel, discourage attorneys from representing defendants, and hurt the practice of law as a profession. The ABA could also make a modified officer of the court argument whereby they are encouraged to seek justice. Such an argument would likely draw more from a rights-based deep theory perhaps along the lines of Locke arguing that all men are born with certain natural rights.

Starting Your Own Practice [Scope, Fees, Supervis., Independ., Comm., Advert., Contact, Name]
Involves Preamble, MR 1.2, 1.5, 5.1, 5.4, 7.1, 7.2, 7.3, 7.4 and 7.5 Involves Bluebook: Holmes, H.L.A. Hart. Scope of representation? The first issue would be whether or not to include/allow for pro bono work at our new firm. The ABA Preamble explicitly states that a lawyer should devote professional time and resources and use civic influence to ensure equal access to our system of justice for all those who cannot afford or secure adequate legal counsel. But this is more of an aspirational goal and not a command. As members of the Mass. Bar we are all aware that the Bar will take a small payment in lieu of performing a certain number of pro bono hours. Even then, however, it is still an aspirational request and there would be no punishment for failing to perform the pro bono hours or for failing to submit payment. Given the great costs involved with starting a law practice, I would initially suggest that we refrain from doing pro bono work. Moreover, following a purely

positivist point of view, the pro bono request is not a valid law since it is not backed by sanctions. Moreover, as HLA Hart pointed out confusing personal values and law is bad philosophy and bad jurisprudence. Fees? The next issue would be how to determine and charge an appropriate fee for our legal services. MR 1.5(a) states that an attorney shall not make an arrangement for, charge, or collect an unreasonable fee or an unreasonable amount for expenses. The factors to be considered under MR 1.5(a) are: (1) the time and labor required, the novelty and difficulty of the questions involved, and the skill requisite to perform the legal service properly; (2) the likelihood, if apparent to the client, that the acceptance of the particular employment will preclude other employment by the lawyer; (3) the fee customarily charged in the locality for similar legal services; (4) the amount involved and the results obtained; (5) the time limitations imposed by the client or by the circumstances; (6) the nature and length of the professional relationship with the client; (7) the experience, reputation, and ability of the lawyer or lawyers performing the services; and (8) whether the fee is fixed or contingent. We would also make sure that the scope of our representations and the fees and expenses were properly understood and consented to in writing to satisfy MR 1.5(b). Depending on the type of legal work we would be doing, we could consider taking a client on a contingent fee basis but not in domestic relations matters or in a criminal case to satisfy MR 1.5(d), and it would have to spell out all the details regards percentages and expenses to satisfy MR 1.5(c). If we split the work with another firm we would make sure that the division of fees was proportional, that the client agreed to it beforehand and that the total fee was reasonable to remain in line with MR 1.5(e). Responsibilities? To satisfy MR 5.1, we would make reasonable efforts to ensure that the firm has in effect measures giving reasonable assurance that all lawyers in the firm conform to the MRCP. Professional independence? Since our new firm is being founded by three attorneys, we would not have to worry about forming a partnership with a non-lawyer in violation of MR 5.4(b). If money got tight however, we would have to search out for other attorneys to contribute and join the firm to satisfy MR 5.4(d). We would not have to worry about violating MR 5.4(a) when we set up our retirement compensation plan even if it included a few non-lawyer paralegals. We would however have to worry about taking on clients whose bills were being paid for by a third-party. We would have to make it clear to the third-party that we are the clients attorney and not their attorney and it is up to the client to make the decisions. MR 5.4(c). Communications concerning our services? If we advertise our services, not only would it have to be in accordance with MR 7.3, but we would have to make sure that we do not make any false or misleading communications about a lawyer or the lawyers services. We would have to make sure that we do not make a material misrepresentation of fact or law, or omit a fact necessary to make the statement considered as a whole not materially misleading to satisfy MR 7.1. Advertising our firm? We would be able to advertise our law firm through written, recorded or electronic communication, including public media so long as it was in accordance with MR 7.1/7.3. We would have to make sure that we are paying reasonable costs of advertisements or paying usual charges for a legal service plan to satisfy MR 7.2(b). We would not simply give away things of value to persons recommending our firm, with the exceptions noted. If we engaged in referring clients pursuant to an agreement, we would have to make sure that the agreement is reciprocal and that the clients are informed about the agreement to satisfy MR 7.2(b)(4). Lastly, any communication about our firm would have to include the name and address of at least one lawyer responsible for the content. Direct contact with prospective clients? We would have to refrain from making in-person, live telephone or real-time electronic solicitations from prospective clients where a significant motive is for pecuniary gain. MR 7.3. However, we could contact other lawyers or persons who have a family, close personal, or prior professional relationship with one of our lawyers. We would also not be able to solicit professional employment from a prospective client if the prospective client makes it known that he or she does not wish to be solicited or if the solicitation involves coercion, duress, or harassment MR 7.3(b). Lastly, if we did mail out any solicitations we would be sure to mark the outside of the envelope with Advertising Material and if by recording, inform them at the beginning and end. MR 7.3(c). Communicating fields of practice or specialization? We would be allowed to communicate that we do or do not practice in certain fields, and if we had a patent or admiralty practice we could also use certain designations. MR 7.4(a), (b) and (c). However, we could not state or imply that a lawyer was certified as a specialist unless the lawyer had been certified by an appropriate state authority or an authority accredited by the ABA and such authority is clearly identified in the communication. If we said that we were simply a specialist or that we specialize in a particular practice, we would be subject to the same false and misleading standard set out in MR 7.1. Moreover, in Mass. we would be held to the higher expert level of knowledge if we were to be sued for malpractice. Firm name and letterhead? Lastly, we would have to have to avoid using a false or misleading firm name and ensure that it did not imply a connection with any governmental agency or public or charitable service. If we had attorneys from other states in our firm and we all used the same letterhead, we would have to indicate which lawyers were admitted to practice in which state. We also could not use the name of any lawyer that is in public office if they are not actively and regularly practicing with the firm. Lastly, we could only state that we were a partnership or otherwise if in fact we were so.

Replacing Moral Turpitude with Criminal Act that Reflect Adversely [Misconduct] Involves MR 8.4 and Comment 2, DR-1-102 Involves Bluebook: Lon Fuller, H.L.A. Hart Is this a step forward? I fully believe that the ABAs adoption of the new language in MR 8.4(b) is a step
forward in the right direction. The previous language contained in the ABA Model Code was too broad and too vague. How does one define moral turpitude and who is to decide whether an act is immoral? Moreover, what if one does not believe in subjective or objective morality? It is too broad because it may apply to criminal acts that have no apparent relationship to ones fitness to practice law. It is too vague because it has been inconsistently interpreted and it cannot be determined with sufficient certainty to which criminal acts it applies. Indeed, even the courts have had difficulty in determining exactly what acts are captured by the phrase moral turpitude. In Jordan v. De George, 341 U.S. 223, 234 (1951), Justice Jacksons dissent speaks to directly to this point. Justice Jackson states how the Court could not provide a clear definition of moral turpitude. He states that even going to the dictionaries, the last resort of the baffled judge, provides little clarification other than defining it as immoral. The Government conceded the point that the term itself provided no exact test for defining what are and are not acts of moral turpitude. The Government then suggested that degrees of severity of the crime could provide an adequate measure by which one could judge whether an act constituted moral turpitude. In a fitting response, Justice Jackson replied that seriousness could not provide such a measure because all offenses denounced by Congress, prosecuted by the Executive, and convicted by the courts, must be deemed in some degree serious' or law enforcement would be a frivolous enterprise. The Government then suggested the moral standards that prevail in contemporary society as a sufficiently definite standard of conduct that is immoral. But once again, Justice Jackson was not persuaded and stated that in a country so large and diverse, How should we ascertain the moral sentiments of masses of persons on any better basis than a guess? Or in the words of Judge Learned Hand, cited by Justice Jackson, Even though we could take a poll, it would not be enough merely to count heads, without any appraisal of the voters. A majority of the votes of those in prisons and brothels, for instance, ought scarcely to outweigh the votes of accredited churchgoers. Nor can we see any reason to suppose that the opinion of clergymen would be a more reliable estimate than our own. Schmidt v. United States, 177 F.2d 450, 451 (2d Cir. 1949). One can see in both Justice Jackson and Judge Hands comments the implicit congruity with H.L.A. Harts belief that confusing personal values and law is bad philosophy and bad jurisprudence. No one would deny that since the legal profession is self-regulating, there must be some standard upon which an individuals can be judged to either within or outside the bounds of acceptable behavior. But defining this standard gets to the heart of the distinction between the Model Codes language and the Model Rules language. As stated by Lon Fuller, a failure to achieve rules at all, so that every issue must be decided on an ad hoc basis, a failure to make rules understandable and a failure of congruence between the rules announced and their actual administration, all present in the Model Codes language, are three of the potential pitfalls in rulemaking. How can one plan his or her conduct if he or she is unaware of what exactly is prohibited? Though not perfect, at least the hypothetical attorney is given some guidance under the Model Rules that MR 8.4 will be violated if he or she engages in a criminal act that reflects adversely on his or her honesty, trustworthiness or fitness as a lawyer in other respects. MR 8.4 provides a rule that is more understandable to both the lawyer and the disciplinary committee enforcing it. In contrast to the Model Codes moral turpitude language, which seemingly required a lawyer to have irreproachable moral standards e.g., EC 1-5 which states that a lawyer should refrain from morally reprehensible conduct the Model Rules language is focused more specifically on only those moral traits necessary to carry out specific professional responsibilities. Moreover, whereas the language of the Model Code and its enforcement centered on the morality or lack thereof of the act itself, the Model Rules are more directed on how the act in question bears on the lawyer's professional integrity. Taken in sum, the ABAs decision to change the language of the Model Code regarding lawyer misconduct into its current form in the Model Rules represents a wise step forward for the legal profession. MR 8.4(b) presents a clearer standard, though not yet perfect, that attorneys may use in planning their actions. It presents a clearer standard for disciplinary committees to use in determining if a violation has occurred. And it also removes the subjective moral characteristic out of the standard providing greater clarity and an inherent fairness to those in the legal profession. In a country that professes its love for individualism and equality, it would seem patently unfair if the legal profession were to subject its membership to the moral standards of one committee of individuals. As H.L.A. Hart stated, It cannot seriously be disputed that the development of law, at all times and places, has in fact been profoundly influenced both by the conventional morality and ideals of particular social groups, and also by forms of enlightened moral criticism urged by individuals, whose moral horizon has transcended the morality currently acceptedBut is does not follow that the criteria of legal validity of particular laws used in a legal system must include, tacitly if not explicitly, a reference to morality or justice.

In-House Lawyer Switching Sides (based on Hull v. Celanese) [Loyalty to Past Clients]
Involves MR 1.6, 1.8, 1.9 and 1.10 Involves Bluebook: St. Thomas Aquinas Should plaintiffs counsel be disqualified? Yes. When determining whether to disqualify an attorney from representing a party, the court need not inquire whether the lawyer did, in fact, receive confidential information. Rather, where it can reasonably be said that in the course of the former representation the attorney might have acquired information related to the subject matter of his subsequent representation, it is the court's duty to order the attorney disqualified. The breach of confidence would not have to be proved; it is presumed in order to preserve the spirit of the Model Rules. The preservation of public trust both in the scrupulous administration of justice and in the integrity of the bar is paramount. Recognizably important are a client's right to counsel of her choice and the consideration of the judicial economy which could be achieved by trying claims in one lawsuit. These considerations must yield, however, to considerations of ethics, which run to the very integrity of our judicial process. In the case, attorney X worked as in-house counsel for Company and as such had a continuing duty of loyalty and confidentiality to her former employer. This is distinctly different from a situation where an attorney might represent a client who is adverse to a former client on a substantially related matter. In the present case, the matter at issue is not merely related to a former representation but it is in fact the same issue. MR 1.9 speaks directly to this issue and generally states that an attorney shall not use information relating to the representation to the disadvantage of the former client. Since the attorney was the in-house counsel for Company on the issue at hand, she would have clearly been privy to information protected by MR 1.6. Moreover, it was very likely information that would not have been generally known or available to the public. As such, the new law firm should have declined to allow her to intervene into their ongoing litigation, as the possible conflicts of interest were readily apparent. MR 1.10 also speaks directly to this point. As the court noted, the sheer possibility that attorney X may have inadvertently divulged Companys confidential information to the new firm is reason enough to disqualify the firm. It is "a necessary and desirable remedy to enforce the lawyer's duty of absolute fidelity and to guard against the danger of inadvertent use of confidential information. Moreover, the prior litigation should not be allowed to use any confidential information that attorney X learned while employed as in-house counsel for Company. Should the attorney be allowed to file suit? A cursory reading of MR 1.8 would suggest that the attorney would not be able to use information she learned during the course of her employment in a suit against her former employer. MR 1.8 states that a lawyer shall not use information relating to representation of a client to the disadvantage of the client unless the client gives informed consent, except as permitted or required by Rules. The key language is the exception. An exception to MR 1.6 allows an attorney to reveal information to the extent reasonably necessary to establish a claim or defense in a controversy between the lawyer and the client. In this case, attorney X could probably argue for disclosure under the policy for self-defense and fee collection. She is entitled to bring suit against her former employer just as any other employee would be allowed. I think that attorney could also, on a natural rights ground invoking St. Thomas Aquinas, argue that it would be an unjust law if attorney was prohibited from vindicating her own rights simply because of her profession. An unjust law is no law at all.

II. Regulating the Profession A. THE SOURCES OF PROFESSIONAL ETHICS TODAY Many states have adopted variants of the ABA models, but often the federal courts in the same state may follow a different version. Courts interpret the rules in three very different contexts: Disciplinary actions initiated by a bar counsel (prosecutor appointed by the state supreme court), heard by a hearing committee (appointed by a board of bar overseers), and appeals from the decisions go to the courts. Disqualification motions usually involve conflict of interest allegations; disqualified attorney may forfeit all fees for the work for the client to date. Malpractice motions courts often look at rules as evidence of the minimum standards of care owed to clients in malpractice cases (though in theory, the rules are for disciplinary actions and are not designed to establish standard of care for tort actions). The ALI Restatement of the Law Governing Lawyers and other Professional Ethics treatises are authoritative sources the courts look to them for guidance, but they are not binding and have no direct legal effect.

B. DEEP THEORIES Deep theories explain humans core motivation for obeying rules. Goal-based deep theories: Focus solely on political or economic outcomes. All laws that, in net effect, promote these ends are legitimate and good. However, goal-based theories are inherently unsatisfactory for those concerned about the legitimacy of the goals and the means employed in pursuit of said goals. Rights-based deep theory: Focused on human freedom. Excellent for defining the parameters of personal freedoms, but they are less helpful in making critical choices within our own area of freedom. Rawls postulates two rules: (i) every person must have the largest political liberty compatible with like liberty for all and (ii) that inequalities in power, wealth, income and other resources must not exist except insofar as they work to the absolute benefit of the worst-off members of society. Duty-based deep theories: Founded on the great classical and religious traditions and include the key tenet that good acts do not necessarily lead to good results (at least not in this life). C. THE HISTORICAL AND MODERN CONTEXT The Attorney Ideal Stepping in the shoes of your client for legal purposes. Client makes the decision. Must tell client what his rights and obligations are under the law. The Officer of the Court Ideal The Court authorizes and supervises attorneys. Certain duties and obligations to the justice system itself. Some suggest that lawyers have a duty to seek a just result, regardless of whether or not it is in the best interest of their client. The Guild Ideal Controlled by their own professional values. Can sometimes conflict with the interests of government or with its clients. Do not want the lawyers to become part of the government apparatus. Generally protects the practice of law as a profession. D. WHO SHOULD MAKE THE RULES GOVERNING LAWYERS IN FEDERAL MATTERS Wide disparity in the stated rules that govern the professional responsibilities of lawyers in the 94 federal district courts of the United States. The standards of attorney conduct in the federal courts are not uniform horizontally across the federal system and are uniform vertically within the federal and state systems in some jurisdictions but not in others. McDade Amendment Makes government attorneys subject to State laws and rules, and local Federal Court rules, governing attorneys in each State where such attorney engages in that attorneys duties, to the same extent and in the same manner as other attorneys in that State. The statute subjects government attorneys to both state laws and rules and federal court rules without explaining how to reconcile these provisions when they conflict. Congress could enact a single set of rules of attorney conduct applicable to all courts. Advantage: The growing nationalization and internationalization of legal practice is hampered by the multiplicity of different rules governing attorney conduct in different states; would make life easier for the DOJ. Disadvantage: Some think our federal systems diffusion of power is good and important states dont want to lose their power to regulate the profession. Plus, Congress does not have time to deal thoughtfully with all the nuances that need to be considered in adjusting the interest of clients, lawyers, and the system of justice and to keep adjusting them as problems appear.

The state judiciaries could organize themselves and create a group that would take greater control of the lawmaking process with regard to attorney conduct the ABA could act in an advisory capacity instead of being in charge of revising the rules. Kaufman thinks the path to successful rulemaking in the area of attorney conduct lies in effective cooperative action of the federal and state judiciaries state courts should figure out how to include the federal courts in their jurisdiction in the process of making the rules AND the power of the ABA should be reduced to true advising.

E. TOWARD AN ECCLESIASTICAL PROFESSIONAL ETHICS Lawyers codes moved from broad canons (1908), to a code (1969), to rules (1983) and at the same time, the focus of these codifications changed from a broad articulation of ethics (1908), to responsibility (1969), to a specific focus on conduct (1983). Ethical Awareness. Rules require initial cognitive awareness of their possible application and ethical awareness to understand the value behind the rules this ethical awareness flows from the underlying moral values or principles that are the foundation of the rule. Role-differentiated behavior the notion that lawyers and priests and others who function in a professional role have specific obligations that flow from their professional roles. The professional obligation is grounded in an assessment that the greater good is achieved by maintaining confidentiality. Professional Codes. Professional codes cannot function in isolation. To be effective, a code must interact and be harmonious with the aspirational goals of the systems in which the professional functions. F. ABA MODEL RULES: A LAWYERS RESPONSIBILITY A lawyer, as a member of the legal profession, is a representative of clients, an officer of the legal system and a public citizen having special responsibility for the quality of justice. In all professional functions a lawyer should be competent, prompt and diligent. As a public citizen, a lawyer should seek improvement of the law, access to the legal system, the administration of justice and the quality of service rendered by the legal profession. A lawyer should further the publics understanding of and confidence in the rule of law and the justice system because legal institutions in a constitutional democracy depend on popular participation and support to maintain their authority. A lawyer should devote professional time and resources and use civic influence to ensure equal access to our system of justice for all those who cannot afford or secure adequate legal counsel. The Rules do not, however, exhaust the moral and ethical considerations that should inform a lawyer, for no worthwhile human activity can be completely defined by legal rules. The Rules simply provide a framework for the ethical practice of law.

III. Loyalty to Present Clients: Concurrent Conflicts of Interest A. MODEL RULE 1.7 CONFLICT OF INTEREST: CONCURRENT CLIENTS (a) Except as provided in paragraph (b), a lawyer shall not represent a client if the representation involves a concurrent conflict of interest. A concurrent conflict of interest exists if: (1) The representation of one client will be directly adverse to another client; or (2) There is a significant risk that the representation of one or more clients will be materially limited by the lawyer's responsibilities to another client, a former client or a third person or by a personal interest of the lawyer. (b) Notwithstanding the existence of a concurrent conflict of interest under paragraph (a), a lawyer may represent a client if: (1) The lawyer reasonably believes that the lawyer will be able to provide competent and diligent representation to each affected client;

(2) The representation is not prohibited by law; (3) The representation does not involve the assertion of a claim by one client against another client represented by the lawyer in the same litigation or other proceeding before a tribunal; and (4) Each affected client gives informed consent, confirmed in writing.

Even if MR 1.7 is triggered, the lawyer can still continue to represent each affected client if the two-pronged test of MR 1.7(b) is met. The first prong requires the informed consent of each client in writing. The second prong is called the objective or reasonableness prong because even if the client consents in writing, the concurrent representation is not permitted unless the lawyer will be able to provide competent and diligent representation to each affected client. A lawyer may take inconsistent legal positions in different tribunals at different times on behalf of different clients. However, a conflict of interests exists if there is a significant risk that a lawyers action on behalf of one client will materially limit the lawyers effectiveness in representing another client in a different case. A client who has given consent to a conflict may revoke the consent, and, like any other client, may terminate the lawyers representation at any time. The McCourt Company v. FPC Properties et al. Facts: Parker Coulter represents McCourt as plaintiffs counsel in action against FPC. Defendants (FPC) brought action to disqualify Parker Coulter from representing McCourt in a lawsuit over property in the Fort Point Channel. FPC is a wholly owned subsidiary of CCF. Parker Coulter has defended CCF (parent company) in several personal injury cases. Four cases were still pending at the time of this lawsuit. The insurer (Aetna) selected Parker Coulter as counsel for CCF. Parker Coulter wants an exception to the general rule (that requires consent from both parties) where a commercial corporation is involved. Holding: The court held that it was irrelevant that the lawsuits were unrelated in subject matter and that it appeared probable that client A would not in fact be prejudiced by the concurrent participation of the law firm in both actions. The undivided loyalty that a lawyer owes to his clients forbids him, without the clients consent, from acting for client A in one action and at the same time against client A in another. Moreover, there is no distinction between clients who are natural person and clients that are corporations. Reasoning: Although McCourt had an interest in choosing counsel it desired; its interests were not absolute. In the absence of consent from CCF, Parker Coulter cannot represent McCourt in action against FPC. Baldasarre v. Butler Facts: Attorney represented the buyer and seller in a complex commercial real estate transaction. Attorney had the consent of both parties. Holding: Court recognizes the need for a new bright line rule prohibiting dual representation in complex commercial transactions. An attorney may not represent both the buyer and the seller in a complex commercial real estate transaction even if both parties give their informed consent. In Re Cohen Facts: The attorney's firm undertook representation of a non-profit company in a trademark proceeding before the U.S. Patent and Trademark Office (PTO). Although the senior attorney, a partner in the law firm, conferred on occasion with the client, it was the attorney's son, an associate in the firm, who performed much of the day-to-day work. During the pendency of the matters before the PTO, the relationship between two of the companies' representatives deteriorated. The firm began representing them both. At the request of one representative, the junior attorney filed an application to withdraw the initial trademark application. This would enable that representative to register the trademark for his own benefit. The other representative was not notified. The Board found that the senior attorney had not met his supervisory responsibility for the performance of the junior attorney and recommended a 30-day suspension. Holding: The court adopted the recommendation, finding that the Board had established a violation of Rule 5.1(c)(2) because the attorney reasonably should have known of the withdrawal application and should have been able to take reasonable remedial action to avoid its consequences.

B. MODEL RULE 1.8 CONFLICTS OF INTERESTS CURRENT CLIENTS SPECIFIC RULES (a) A lawyer shall not enter into a business transaction with a client or knowingly acquire an ownership, possessory, security or other pecuniary interest adverse to a client unless: (1) The transaction and terms on which the lawyer acquires the interest are fair and reasonable to the client and are fully disclosed and transmitted in writing in a manner that can be reasonably understood by the client; (2) The client is advised in writing of the desirability of seeking and is given a reasonable opportunity to seek the advice of independent legal counsel on the transaction; and (3) The client gives informed consent, in a writing signed by the client, to the essential terms of the transaction and the lawyer's role in the transaction, including whether the lawyer is representing the client in the transaction. (b) A lawyer shall not use information relating to representation of a client to the disadvantage of the client unless the client gives informed consent, except as permitted or required by Rules. (c) A lawyer shall not solicit any substantial gift from a client, including a testamentary gift, or prepare on behalf of a client an instrument giving the lawyer or a person related to the lawyer any substantial gift unless the lawyer or other recipient of the gift is related to the client. For purposes of this paragraph, related persons include a spouse, child, grandchild, parent, grandparent or other relative or individual with whom the lawyer or the client maintains a close, familial relationship. (d) Prior to the conclusion of representation of a client, a lawyer shall not make or negotiate an agreement giving the lawyer literary or media rights to a portrayal or account based in substantial part on information relating to the representation. (e) A lawyer shall not provide financial assistance to a client in connection with pending or contemplated litigation, except that: (1) A lawyer may advance court costs and expenses of litigation, the repayment of which may be contingent on the outcome of the matter; and (2) A lawyer representing an indigent client may pay court costs and expenses of litigation on behalf of the client. (f) A lawyer shall not accept compensation for representing a client from one other than the client unless: (1) The client gives informed consent; (2) There is no interference with the lawyer's independence of professional judgment or with the client-lawyer relationship; and (3) Information relating to representation of a client is protected as required by Rule 1.6. (g) A lawyer who represents two or more clients shall not participate in making an aggregate settlement of the claims of or against the clients, or in a criminal case an aggregated agreement as to guilty or nolo contendere pleas, unless each client gives informed consent, in a writing signed by the client. The lawyer's disclosure shall include the existence and nature of all the claims or pleas involved and of the participation of each person in the settlement. (h) A lawyer shall not: (1) Make an agreement prospectively limiting the lawyer's liability to a client for malpractice unless the client is independently represented in making the agreement; or (2) Settle a claim or potential claim for such liability with an unrepresented client or former client unless that person is advised in writing of the desirability of seeking and is given a reasonable opportunity to seek the advice of independent legal counsel in connection therewith. (i) A lawyer shall not acquire a proprietary interest in the cause of action or subject matter of litigation the lawyer is conducting for a client, except that the lawyer may: (1) Acquire a lien authorized by law to secure the lawyer's fee or expenses; and (2) Contract with a client for a reasonable contingent fee in a civil case. (j) A lawyer shall not have sexual relations with a client unless a consensual sexual relationship existed between them when the client-lawyer relationship commenced. (k) While lawyers are associated in a firm, a prohibition in the foregoing paragraphs (a) through (i) that applies to any one of them shall apply to all of them.

10

C. THE LEGAL ETHICS OF LOUIS D. BRANDEIS The Warren Matter If it occurred today, it would be a clear violation of MR 1.7. Interest of beneficiaries was directly adverse to the lessees of the beneficiaries factories not reasonable for one attorney to represent both interests. Lawyers dont counsel situations, they counsel clients. The Lennox Matter If it occurred today, it would be a clear violation of MR 1.7. Should have known that he was representing a creditor (adverse party). The United Shoe Matter If it occurred today, it would not violate the Model Rules, unless it was the same tribunal or the same matter. Represents the guild ideal. But, when a practitioner is in doubt on an ethical question, the best answer is usually NO, and perhaps Brandeis should have given that answer when the reformers ardor seized him to tilt against [his former clients position]. IV. Loyalty to Past Clients: Successive and Imputed Conflicts of Interest A. LEGAL FICTIONS Imputed conflict of interest is a legal fiction. They occur whenever we assume something is true, for theoretical or policy reasons, when we know, in fact, that it may not be true at all. Because of the potential danger, the drafters built in important exceptions to the Model Rules. First, imputed disqualification only occurs when the lawyer had acquired information protected by Rule 1.6 and 1.9(c) that is material to the matter. Second, it has to be the same or substantially related matter, and the new clients interests must be materially adverse to the interests of the former client. Third, the former client can always remove bar by giving informed consent in writing. B. MODEL RULE 1.9 DUTIES TO FORMER CLIENTS (a) A lawyer who has formerly represented a client in a matter shall not thereafter represent another person in the same or a substantially related matter in which that person's interests are materially adverse to the interests of the former client unless the former client gives informed consent, confirmed in writing. (b) A lawyer shall not knowingly represent a person in the same or a substantially related matter in which a firm with which the lawyer formerly was associated had previously represented a client: (1) Whose interests are materially adverse to that person; and (2) About whom the lawyer had acquired information protected by Rules 1.6 and 1.9(c) that is material to the matter; unless the former client gives informed consent, confirmed in writing. (c) A lawyer who has formerly represented a client in a matter or whose present or former firm has formerly represented a client in a matter shall not thereafter: (1) Use information relating to the representation to the disadvantage of the former client except as these Rules would permit or require with respect to a client, or when the information has become generally known; or (2) Reveal information relating to the representation except as these Rules would permit or require with respect to a client. Information that has been disclosed to the public or to other parties adverse to the former client ordinarily will not be disqualifying. On the other hand, knowledge of specific facts gained in a prior representation that are relevant to the matter in question ordinarily will preclude such a representation. If a lawyer while with one firm acquired no knowledge or information relating to a particular client of the firm, and that lawyer later joined another firm, neither the lawyer individually nor

11

the second firm is disqualified from representing another client in the same or a related matter even though the interests of the two clients conflict. Emle Industries, Inc. v. Patentex, Inc. Facts: Attorney Rabin represented Burlington in an earlier patent suit that called into question the nature and scope of Burlingtons control over Patentex. Rabin now represented a group of plaintiffs in a case against Patentex. Holding: The Court of Appeals affirmed the lower courts decision that Rabins earlier defense of Burlington against charges that it controlled Patentex for illegal purposes precluded him from pressing similar claims in the instant suit. Reasoning: Each proceeding involved a claim that Burlington controlled Patentex and used that control for an illegal purpose, and therefore the claims were substantially related. The concern is enforcing the lawyers duty of absolute fidelity and the spirit of the rule relating to privileged communications. A lawyers good faith, although essential in all his professional activity, is nevertheless, an inadequate safeguard when standing alone. Even the most rigorous self-discipline might not prevent a lawyer from unconsciously using or manipulating a confidence acquired in the earlier representation and transforming it into a telling advantage in the subsequent litigation. However, because the claims were substantially related, there was no need to inquire whether Rabin in fact had access to confidential information when he represented Burlington in the earlier case. C. MODEL RULE 1.10 IMPUTATION OF CONFLICTS OF INTEREST (a) While lawyers are associated in a firm, none of them shall knowingly represent a client when any one of them practicing alone would be prohibited from doing so by Rules 1.7 or 1.9, unless: (1) The prohibition is based upon a personal interest of the prohibited lawyer and does not present a significant risk of materially limiting the representation of the client by the remaining lawyers in the firm; or (2) The prohibition is based upon Rule 1.9(a), (or (b)), and (i) The disqualified lawyer is timely screened from any participation in the matter and is apportioned no part of the fee therefore; (ii) Written notice is promptly given to any affected former client to enable the former client to ascertain compliance with the provisions of this Rule, which shall include a description of the screening procedures employed; a statement of the firm's and of the screened lawyer's compliance with these Rules; a statement that review may be available before a tribunal; and an agreement by the firm to respond promptly to any written inquiries or objections by the former client about the screening procedures; and (iii) Certifications of compliance with these Rules and with the screening procedures are provided to the former client by the screened lawyer and by a partner of the firm, at reasonable intervals upon the former client's written request and upon termination of the screening procedures. (b) When a lawyer has terminated an association with a firm, the firm is not prohibited from thereafter representing a person with interests materially adverse to those of a client represented by the formerly associated lawyer and not currently represented by the firm, unless: (1) The matter is the same or substantially related to that in which the formerly associated lawyer represented the client; and (2) Any lawyer remaining in the firm has information protected by Rules 1.6 and 1.9(c) that is material to the matter. (c) A disqualification prescribed by this rule may be waived by the affected client under the conditions stated in Rule 1.7. (d) The disqualification of lawyers associated in a firm with former or current government lawyers is governed by Rule 1.1. Silver Chrysler Plymouth, Inc. v. Chrysler Motors Corporation Facts: Attorney Schreiber was an associate at the law firm of Kelley Drye, which handled much of Chryslers business. While at Kelley Drye, Schreiber worked on certain Chrysler matters. Schreiber

12

quit Kelley Drye and moved to the law firm of Hammond & Schreiber, which was representing Silver Chrysler in this case. Holding: The Court of Appeals agreed with the lower court that refusal to disqualify Schreiber and his firm would not create an appearance of impropriety. Reasoning: Guarding against the inadvertent use of confidential information does not require such a blanket approach. As opposed to other cases in which the attorneys at issue were heavily involved in the earlier cases, here Schreiber was only involved in minor elements of Chrysler matters. [T]here was ample basis for crediting Schreibers denial of having worked on them and concluding that Schreibers involvement was, at most, limited to brief, informal discussions on a procedural matter or research on a specific point of law. Under these circumstances, Schreibers role could not be considered representation within the meaning of Emle so as to require disqualification. It is unquestionably true that in the course of their work at large law firms, associates are entrusted with the confidences of some of their clients. But it would be absurd to conclude that immediately upon their entry on duty they become the recipients of knowledge as to the names of all the firms clients, the contents of all files relating to such clients, and all confidential disclosures by client officers or employees to any lawyer in the firm. Neither Chrysler nor any other client of a law firm can reasonably expect to foreclose either all lawyers formerly at the firm or even those who have represented it on unrelated matters from subsequently representing an opposing party. Hull v. Celanese Corporation Facts: Delulio was an attorney on the corporate legal staff of Celanese. Plaintiff Joan Hull brought a Title VII sex-based discrimination suit against Celanese using the law firm of Rabinowitz, Boudin & Standard. Delulio switched sides after meeting Hull socially and brought a similar Title VII with Hull against Celanese. Celanese opposed the proposed intervention and additionally sought the disqualification of the Rabinowitz firm based on the risk that confidential information received by Delulio as Celaneses attorney might be used by Rabinowitz against Celanese. Holding: The Court of Appeals affirmed the trial courts decision that the Rabinowitz firm should be disqualified because of the chance of improper disclosure of confidential information from Delulio that might help the Hull-Delulio joint case. Reasoning: In a disqualification situation, any doubt is to be resolved in favor of disqualification. This is, in short, one of those cases in which disqualification is a necessary and desirable remedyto enforce the lawyers duty of absolute fidelity and to guard against the danger of inadvertent use of confidential information. The Rabinowitz firm had notice that Delulio had worked on the defense of the Hull case and should have declined representation when approached. In granting a motion to disqualify an attorney, courts recognize three competing interests: (i) the client's interest in freely selecting counsel of her choice, (ii) the adversary's interest in the trial free from the risk of even inadvertent disclosures of confidential information and (iii) the public's interest in the scrupulous administration of justice. Analytica, Inc. v. NPD Research, Inc. Facts: John Malec worked for NPD, and was going to receive an additional 10 percent of the company. He hired attorney Fine of the law firm Schwartz & Freeman to structure the deal. NPD gave Fine the information he needed to accomplish his task information on NPDs financial condition, sales trends, and management and Fine billed NPD for his services and NPD paid the bill of $850. Malec left NPD and his wife set up a competing company called Analytica, which retained S&F as its counsel. Analytica brought a suit against NPD charging that NPD was engaged in anticompetitive behavior that was preventing Analytica from establishing itself in the market. NPD moved to disqualify both of the law firms representing Analytica (S& F and the law firm P&H). Holding: The Court of Appeals held that P&H had no standing to appeal; and affirmed the disqualification of S&F because a law firm is not permitted to switch sides if its former representation was substantially related to its new representation, no matter what screens it sets up. Reasoning: The law firm of P&H was disqualified not for anything it did or failed to do but simply because as S&Fs co-counsel it had access, actual or potential, to whatever confidential information S&F had obtained while representing NPD. It didnt matter whether NPD or Malec was the actual client of S&F in the initial matter; all that mattered was S&F had access to confidential information

13

(and even if it did matter, NPD paid all of S&Fs bills and none of the parties retained any other counsel except S&F). D. MODEL RULE 1.11 SPECIAL CONFLICTS OF INTEREST FOR FORMER AND CURRENT GOVERNMENT OFFICERS AND EMPLOYEES (a) Except as law may otherwise expressly permit, a lawyer who has formerly served as a public officer or employee of the government: (1) Is subject to Rule 1.9(c); and (2) Shall not otherwise represent a client in connection with a matter in which the lawyer participated personally and substantially as a public officer or employee, unless the appropriate government agency gives its informed consent, confirmed in writing, to the representation. (b) When a lawyer is disqualified from representation under paragraph (a), no lawyer in a firm with which that lawyer is associated may knowingly undertake or continue representation in such a matter unless: (1) The disqualified lawyer is timely screened from any participation in the matter and is apportioned no part of the fee therefrom; and (2) Written notice is promptly given to the appropriate government agency to enable it to ascertain compliance with the provisions of this rule. (c) Except as law may otherwise expressly permit, a lawyer having information that the lawyer knows is confidential government information about a person acquired when the lawyer was a public officer or employee, may not represent a private client whose interests are adverse to that person in a matter in which the information could be used to the material disadvantage of that person. As used in this Rule, the term "confidential government information" means information that has been obtained under governmental authority and which, at the time this Rule is applied, the government is prohibited by law from disclosing to the public or has a legal privilege not to disclose and which is not otherwise available to the public. A firm with which that lawyer is associated may undertake or continue representation in the matter only if the disqualified lawyer is timely screened from any participation in the matter and is apportioned no part of the fee therefrom. (d) Except as law may otherwise expressly permit, a lawyer currently serving as a public officer or employee: (1) Is subject to Rules 1.7 and 1.9; and (2) Shall not: (i) participate in a matter in which the lawyer participated personally and substantially while in private practice or nongovernmental employment, unless the appropriate government agency gives its informed consent, confirmed in writing; or (ii) negotiate for private employment with any person who is involved as a party or as lawyer for a party in a matter in which the lawyer is participating personally and substantially, except that a lawyer serving as a law clerk to a judge, other adjudicative officer or arbitrator may negotiate for private employment as permitted by Rule 1.12(b) and subject to the conditions stated in Rule 1.12(b). (e) As used in this Rule, the term "matter" includes: (1) Any judicial or other proceeding, application, request for a ruling or other determination, contract, claim, controversy, investigation, charge, accusation, arrest or other particular matter involving a specific party or parties, and (2) Any other matter covered by the conflict of interest rules of the appropriate government agency. General Motors Corporation v. City of New York Facts: Attorney Reycraft formerly worked for the Antitrust Division of the Department of Justice (from 1952-1962), and in that capacity he worked on the investigation of alleged monopolization by GM of the city and intercity bus business, but was never in active charge of the case. The City of New York brought a class action claim against GM in 1972, alleging that GM had violated the antitrust laws principally by monopolizing or attempting to monopolize the nationwide market for city buses. In that second case, Reycraft served as the citys privately-retained counsel on a contingent-fee basis. The two suits were strikingly similar, though perhaps not identical in every respect.

14

Holding: The Court of Appeals concluded that Reycrafts disqualification was required to avoid even the appearance of professional impropriety. Reasoning: The court went out of its way to say that Reycraft was not necessarily a bad man, but that the court must act with scrupulous care to avoid any appearance of impropriety lest it taint both the public and private segments of the legal profession. Reycraft and his firms opportunity to earn a substantial fee for Reycrafts services was plainly private employment; the court did not buy Reycrafts pro bono publico characterization of his work, given the potential size of the contingent fee. This is not the kind of situation that would chill the ardor for government service because this case is strikingly similar to Reycrafts earlier case. V. Confidentiality A. TYPES OF INFORMATION Confidential professional information can be roughly divided into (i) information that is subject to the attorney-client privilege and (ii) information that is governed by the professional rules limiting disclosure by a lawyer. Information subject to the evidentiary privilege usually may not be used as evidence. Information governed by the professional rules usually may not be voluntarily disclosed by a lawyer to anyone without client consent, but is usually not protected against courtordered disclosure. In general, the privilege only applies to information communicated between a lawyer and a client in confidence for the purpose of seeking legal advice. Narrow scope. Does not cover information communicated by 3rd party. Does not cover information outside of the scope of legal representation. Communication cannot be made in presence of 3rd party otherwise privilege waived, unless 3rd party is EE of lawyer. In contrast, the confidentiality rule apples not merely to matters communicated in confidence by the client but to virtually all information relating to the representation, whatever its source. Broader scope. Governed by state ethics rules. Restatement (3rd) of Law Governing Lawyers 59 Confidential Client Information Confidential client information consists of information relating to representation of a client, other than information that is generally known. Covers all information relating to representation of a client, whether in oral, documentary, electronic, photographic, or other forms. It covers information gathered from any source, including sources such as third persons whose communications are not protected by the attorney-client privilege. It includes work product that the lawyer develops in representing the client, such as the lawyer's notes to a personal file, whether or not the information is immune from discovery as lawyer work product. It includes information acquired by a lawyer in all client-lawyer relationships, including functioning as inside or outside legal counsel, government or private-practice lawyer, counselor or litigator, advocate or intermediary. It applies whether or not the client paid a fee, and whether a lawyer learns the information personally or through an agent, for example information acquired by a lawyer's partners or associate lawyers or by an investigator, paralegal, or secretary. Information acquired by an agent is protected even if it was not thereafter communicated to the lawyer, such as material acquired by an investigator and kept in the investigator's files. A lawyer may not justify adverse use or disclosure of client information simply because the information has become known to third persons, if it is not otherwise generally known. Moreover, if a current client specifically requests that information of any kind not be used or disclosed in ways otherwise permissible, the lawyer must either honor that request or withdraw from the representation. A lawyer is prohibited from using or disclosing confidential client information if either of two conditions exists risk of harm to the client or client instruction.

15

Restatement (3rd) of Law Governing Lawyers 68 Attorney-Client Privilege (i) Where legal advice of any kind is sought (ii) from a professional legal advisor in his capacity as such, (iii) the communications relating to that purpose, (iv) made in confidence (v) by the client, (vi) are at his instance permanently protected (vii) from disclosure by himself or by the legal advisor, (viii) except the protection be waived. Restatement (3rd) of Law Governing Lawyers 73 Privilege for an Organization When a client is a corporation, unincorporated association, partnership, trust, estate, sole proprietorship, or other for-profit or not-for-profit organization, the attorney-client privilege extends to a communication that: Otherwise qualifies as privileged; Is between an agent of the organization and a privileged person; Concerns a legal matter of interest to the organization; and Is disclosed only to: (a) privileged persons and (b) other agents of the organization who reasonably need to know of the communication in order to act for the organization. B. PHYSICAL EVIDENCE MR 3.4(a) attempts to resolve the balance of a lawyers duty of loyalty and confidentiality by stating that a lawyer shall not unlawfully obstruct or unlawfully alter, destroy or conceal a document or other material having potential evidentiary value. Applicable law may permit a lawyer to take temporary possession of physical evidence of client crimes for the purpose of conducting a limited examination that will not alter or destroy material characteristics of the evidence. However, it may also require a lawyer to turn the evidence over to the police or other prosecuting authority, depending on the circumstances. C. MODEL RULE 1.6 CONFIDENTIALITY OF INFORMATION (a) A lawyer shall not reveal information relating to the representation of a client unless the client gives informed consent, the disclosure is impliedly authorized in order to carry out the representation or the disclosure is permitted by paragraph (b). (b) A lawyer may reveal information relating to the representation of a client to the extent the lawyer reasonably believes necessary: (1) To prevent reasonably certain death or substantial bodily harm; (2) To prevent the client from committing a crime or fraud that is reasonably certain to result in substantial injury to the financial interests or property of another and in furtherance of which the client has used or is using the lawyer's services; (3) To prevent, mitigate or rectify substantial injury to the financial interests or property of another that is reasonably certain to result or has resulted from the client's commission of a crime or fraud in furtherance of which the client has used the lawyer's services; (4) To secure legal advice about the lawyer's compliance with these Rules; (5) To establish a claim or defense on behalf of the lawyer in a controversy between the lawyer and the client, to establish a defense to a criminal charge or civil claim against the lawyer based upon conduct in which the client was involved, or to respond to allegations in any proceeding concerning the lawyer's representation of the client; (6) To comply with other law or a court order. State rel. Sowers v. Olwell Facts: Attorney subpoenaed to turn over knife he held for a client charged with murder. Attorney declined to discuss whether he held a knife for Client Gray. Unclear how lawyer came into possession of the knife, either through an independent investigation, 3rd party or through some communication between Client Gray and lawyer directly. Lawyer found in contempt for refusing to turn over knife. Holding: Regarding the subpoena to testify, the court held that enough evidence existed to believe knife obtained as a result of direct communications between Client Gray and lawyer. Information concerning a crime or fraud communicated by Client to lawyer is protected by the privilege. The

16

Subpoena to testify at coroners inquest similar to subpoena to testify in Court and thus covered by attorney-client privilege. Regarding knife as evidence, the court held that the knife can be withheld by lawyer for a reasonable period of time. Cannot be withheld indefinitely. Attorney should as an officer of the court turn over the knife after a reasonable period of time. The privilege can be preserved even after the knife is turned over to the Court. Prosecution cannot disclose to the jury the source of the evidence. Regarding imputation of Clients Privilege to lawyer, the court held that the lawyer couldnt claim that clients privilege against self-incrimination extends to him as counsel. In Re Ryder Facts: L represented C under investigation for bank robbery. L suspected C involved in robbery and was afraid he might cover evidence up before charged with crime. L called former officer of State Bar Association to discuss the situation, omitting names. L wanted to take money from clients safe deposit box, put it in his own box, and then claim privilege so cops couldnt reach the stolen money but client couldnt dispose of it either. Felt cops would eventually find the money if left in Ls box and it would be returned to the bank. L had C sign power of attorney re: safe deposit box and contents. Did not tell C that money would eventually be returned to the bank. POA simply allowed L to dispose of money as he saw fit. L removed stolen money and sawed off shotgun from Cs box and put it in Ls box. Police obtained search warrant for C and Ls safe deposit boxes. Lawyer Ryder removed as Cs lawyer and charged with ethical violations. Holding: The court held that Ryder be temporarily suspended for participating in this crime. Ryder could easily infer that money had actually been stolen and thus Ryder acted beyond scope of attorneyclient privilege. Ryder initiated the movement of the money and gun. This was more than a mere communication between client and attorney. Reasoning: The court distinguished this case from those where evidence is protected from seizure when in the hands of a client. If evidence cannot be seized from a client, it cannot be seized from the clients attorney. Here, the evidence could be seized from the client, thus attorney has no greater protection. Ryder charged with violations of ethical canons: suppressing evidence of a crime, violating the law, failing to fulfill his obligations to the court, and holding confidential information relating to a future crime. Clients concealing of stolen money from the bank is considered a continuing offense. Thus, lawyer cannot claim confidentiality as regards a continuing offense. Meyerhofer v. Empire Fire and Marine Ins. Co. Facts: Plaintiff shareholders filed class action lawsuit against Empire for failure to disclose unpaid legal fees to their lawyers of $200K in their SEC filing statement. Goldberg was lawyer for Sitomer firm, which represented Empire. He disagreed with firms nondisclosure of this and other material information and later resigned from he firm as a result. Goldberg later named defendant in shareholder suit and asked Sitomer to provide him information to show he knew nothing about an illegal finders fee arrangement. Goldberg provided confidential information re: Empire to plaintiffs in exchange for being dropped as a plaintiff. Empire sought disqualification of Goldberg and plaintiffs firm in this suit because they were privy to confidential information about the opposing party in violation of several Ethical Canons. Holding: The court held that Goldberg was prohibited from engaging in legal representation of plaintiff against his former clients and enjoined him from revealing further confidential information regarding his former client except in trial or deposition. Moreover, the plaintiffs firm was not disqualified from continuing representation because Goldbergs revelation was ethical according to exceptions to confidentiality rules. Reasoning: Goldberg was allowed to reveal confidential client information to protect himself against accusations of wrongful conduct under DR4-101(C). Purcell v. District Attorney for Suffolk District Facts: Public interest lawyer revealed to police his clients intention to burn down his apartment building in response to being evicted. The DA later tried to force Purcell to testify about what his former client had said to him. Holding: The court held that Crime-Fraud exception to privilege doctrine, where an attorney is consulted for the purpose of receiving legal advice or assistance in furtherance of criminal conduct, does not does not apply here client did not seek the lawyers assistance in committing the crime here. He simply discussed wanting to burn down the apartment building as an offhand remark.

17

Reasoning: We dont want to deter lawyers from coming forward with information to protect public safety due to fear of having to testify in court against their clients. Having a broad rule encourages lawyers to discuss future crimes with clients and deter them. A judge can conduct an in camera hearing to decide whether the client-lawyer communication is privileged or fits into the crime-fraud exception. Not enough evidence to even require in camera review here. McClure v. Thompson Facts: Defendant convicted of murder after his attorney anonymously reported the location of his victims bodies to police. The defendant argued that (i) the lawyer failed to obtain his consent before revealing confidential information to a 3rd party; (ii) failed to ask enough questions to determine whether the children might still be alive, therefore triggering an exception to the duty of confidentiality; and (iii) that the lawyer was more concerned with the victims interests than those of his client, thus constituting a conflict of interest. Holding: The court held that it was unclear how evidence of breach of rules of professional ethics should be treated in terms of a 6th Amendment violation (i.e., State ethics rules are guides as to actions of a reasonable lawyer but they are not dispositive). On Issue #1: Client did consent to revealing confidential information, providing lawyer with detailed map of where to find bodies from which he could infer consent. However, consent was not informed, or gained through proper consultation, thus not valid as waiver to confidentiality. On Issue #2: Lawyer was allowed to reveal confidential information because he believed that the children may still be alive and wanted to find them before they died. This exception only allows the attorney to reveal confidences that he reasonably believes necessary to prevent the harm. On Issue #3: No ineffective assistance of counsel. Trying to find children alive would have benefited both children and client. Following an exception to ethical rules to prevent bodily harm is not in conflict with clients interest The standard set forth included an assessment of objective reasonableness: Lawyer must reasonably believe that the harm will occur; lawyer must engage in reasonable investigation and inquiry to determine whether harm will occur; lawyer here had reasonable belief because the client said, Jesus saved the children and never explicitly said they were dead. This is a close call and if reviewed de novo, Court might find differently. Because this is heard on appeal, Court upholds lower courts finding. Dissent: The dissent argued that the court misread the facts completely. The dissent argued that the majority applied a subjective, not objective reasonableness test. The lawyers conduct violated both rules of ethics and 6th Amendment guarantee of effective assistance of counsel. A firm factual basis is the proper standard for judging lawyers disclosure of client confidences. The lawyers behavior was unreasonable because he did not possess sufficient information to infer that the children were still alive and did not conduct a reasonable investigation to determine this. Thus, the lawyers conduct was unreasonable because he acted on bare suspicion alone. D. MASS. RULE 1.6. (a) A lawyer shall not reveal confidential information relating to representation of a client unless the client consents after consultation, except for disclosures that are impliedly authorized in order to carry out the representation, and except as stated in paragraph (b). (b) A lawyer may reveal, and to the extent required by Rule 3.3 [client perjures himself], Rule 4.1(b) [fail to disclose to 3rd person], or Rule 8.3 must reveal [reporting pro. misconduct], such information: (1) to prevent the commission of a criminal or fraudulent act that the lawyer reasonably believes is likely to result in death or substantial bodily harm, or in substantial injury to the financial interests or property of another, or to prevent the wrongful execution or incarceration of another; (2) to the extent the lawyer reasonably believes necessary to establish a claim or defense on behalf of the lawyer in a controversy between the lawyer and the client, to establish a defense to a criminal charge or civil claim against the lawyer based upon conduct in which the client was involved, or to respond to allegations in any proceeding concerning the lawyer's representation of the client; (3) to the extent the lawyer reasonably believes necessary to rectify client fraud in which the lawyer's services have been used, subject to Rule 3.3 (e); (4) when permitted under these rules or required by law or court order.

18

(c) A lawyer participating in a lawyer assistance program, as hereinafter defined, shall treat the person so assisted as a client for the purposes of this rule. Lawyer assistance means assistance provided to a lawyer, judge, other legal professional, or law student by a lawyer participating in an organized nonprofit effort to provide assistance in the form of (a) counseling as to practice matters (which shall not include counseling a law student in a law school clinical program) or (b) education as to personal health matters, such as the treatment and rehabilitation from a mental, emotional, or psychological disorder, alcoholism, substance abuse, or other addiction, or both. A lawyer named in an order of the Supreme Judicial Court or the Board of Bar Overseers concerning the monitoring or terms of probation of another attorney shall treat that other attorney as a client for the purposes of this rule. Any lawyer participating in a lawyer assistance program may require a person acting under the lawyer's supervision or control to sign a nondisclosure form approved by the Supreme Judicial Court. Nothing in this paragraph (c) shall require a bar association-sponsored ethics advisory committee, the Office of Bar Counsel, or any other governmental agency advising on questions of professional responsibility to treat persons so assisted as clients for the purpose of this rule.

E. MODEL RULE 1.6 V. MASS. RULE 1.6 MR 1.6: Covers any information relating to representation of a client. Mass Rule 1.6: Covers only confidential information relating to representation of a client. Narrower than MR 1.6. Addition of confidential excludes information generally known to the public. Exceptions: MR 1.6: (a) may reveal information; (b) to prevent crime or fraud that is reasonably certain; (c) death of substantial bodily injury or financial harm to another; and (d) lawyers services must have been used in furtherance of the crimes in order to trigger the exception. Mass. Rule 1.6: (a) may reveal information; (b) to prevent crime or fraud lawyer reasonably believes; (c) death or substantial bodily injury or financial harm to another; (d) reasonable belief standard. F. MODEL RULE 3.4 FAIRNESS TO OPPOSING PARTY AND COUNSEL A lawyer shall not: (a) Unlawfully obstruct another party's access to evidence or unlawfully alter, destroy or conceal a document or other material having potential evidentiary value. A lawyer shall not counsel or assist another person to do any such act; (b) Falsify evidence, counsel or assist a witness to testify falsely, or offer an inducement to a witness that is prohibited by law; (c) Knowingly disobey an obligation under the rules of a tribunal, except for an open refusal based on an assertion that no valid obligation exists; (d) In pretrial procedure, make a frivolous discovery request or fail to make reasonably diligent effort to comply with a legally proper discovery request by an opposing party; (e) In trial, allude to any matter that the lawyer does not reasonably believe is relevant or that will not be supported by admissible evidence, assert personal knowledge of facts in issue except when testifying as a witness, or state a personal opinion as to the justness of a cause, the credibility of a witness, the culpability of a civil litigant or the guilt or innocence of an accused; or (f) Request a person other than a client to refrain from voluntarily giving relevant information to another party unless: (1) The person is a relative or an employee or other agent of a client; and (2) The lawyer reasonably believes that the person's interests will not be adversely affected by refraining from giving such information.

19

G. MODEL RULE 4.4 RESPECT FOR RIGHTS OF THIRD PERSONS (a) In representing a client, a lawyer shall not use means that have no substantial purpose other than to embarrass, delay, or burden a third person, or use methods of obtaining evidence that violate the legal rights of such a person. (b) A lawyer who receives a document relating to the representation of the lawyer's client and knows or reasonably should know that the document was inadvertently sent shall promptly notify the sender. Rico v. Mitsubishi Motors Corp. Facts: Plaintiffs for car company accidentally received document detailing defenses strategy and case strength and weaknesses. Plaintiffs never notified defense that they held this document but instead used it to impeach defense witnesses on the stand. The plaintiffs lawyer argued that the document was no longer subject to work product privilege once defense expert was called to testify and that the defense waived their right to claim this privilege by not doing so in a timely manner. The defense counsel argued that they did not realize document was their own immediately and plaintiffs did not timely disclose this, thus they shouldnt be punished for not requesting waiver immediately. Holding: The court held that the document was not protected by attorney-client privilege, however the notes were attorney work product. Defense placed at a substantial disadvantage at trial and plaintiffs attorneys disqualified from the case. The plaintiffs counsel had a duty to notify opposing counsel that they received this document. An attorneys impressions, conclusions, opinions, or legal theories is immune from discovery. Similarly, witnesses statements may be discoverable but an attorneys notes on the statements are not. Reasoning: Plaintiffs Ethical Duty If material is obviously privileged or appears confidential or privileged and is received inadvertently, then lawyer may only examine them to the extent necessary to determine whether or not they are privileged and immediately notify the sender of the mistake. Parties may then agree on how to solve the situation or seek judicial intervention. Necessary remedy depends on the amount of damage done. Professional Ethics Comm. of Board of Bar Overseers of the Bar of Maine The Commission concluded that Counsel X may use an inadvertently disclosed privileged memorandum or the information contained in it to the extent permitted by the rules of procedure and evidence, but that he should notify Counsel Y, the sending lawyer, of the fact that the document has been received and provide a copy of the document to Counsel Y upon request. The attorney-client privilege is a shield against deliberate intrusion; it is not an insurer against inadvertent disclosure. Opinion is contrary to the ABA Standing Comm. on Ethics and Prof. Resp. V. Truthfulness A. MODEL RULE 3.3 CANDOR TOWARD THE TRIBUNAL (a) A lawyer shall not knowingly: (1) make a false statement of fact or law to a tribunal [could include a commission, a committee, a board of review, etc] or fail to correct a false statement of material fact or law previously made to the tribunal by the lawyer; (2) fail to disclose to the tribunal legal authority in the controlling jurisdiction known to the lawyer to be directly adverse to the position of the client and not disclosed by opposing counsel; or (3) offer evidence that the lawyer knows to be false. If a lawyer, the lawyer's client, or a witness called by the lawyer, has offered material evidence and the lawyer comes to know of its falsity, the lawyer shall take reasonable remedial measures, including, if necessary, disclosure to the tribunal. A lawyer may refuse to offer evidence, other than the testimony of a defendant in a criminal matter that the lawyer reasonably believes is false. (b) A lawyer who represents a client in an adjudicative proceeding and who knows that a person intends to engage, is engaging or has engaged in criminal or fraudulent conduct related

20

to the proceeding shall take reasonable remedial measures, including, if necessary, disclosure to the tribunal. (c) The duties stated in paragraphs (a) and (b) continue to the conclusion of the proceeding, and apply even if compliance requires disclosure of information otherwise protected by Rule 1.6. (d) In an ex parte proceeding, a lawyer shall inform the tribunal of all material facts known to the lawyer that will enable the tribunal to make an informed decision, whether or not the facts are adverse.

If a lawyer knows that the client intends to testify falsely or wants the lawyer to introduce false evidence, the lawyer should seek to persuade the client that the evidence should not be offered. If the persuasion is ineffective and the lawyer continues to represent the client, the lawyer must refuse to offer the false evidence. If only a portion of a witness's testimony will be false, the lawyer may call the witness to testify but may not elicit or otherwise permit the witness to present the testimony that the lawyer knows is false. The prohibition against offering false evidence only applies if the lawyer knows that the evidence is false. A lawyer's reasonable belief that evidence is false does not preclude its presentation to the trier of fact. A lawyer's knowledge that evidence is false, however, can be inferred from the circumstances. Thus, although a lawyer should resolve doubts about the veracity of testimony or other evidence in favor of the client, the lawyer cannot ignore an obvious falsehood. Nix v. Whiteside Facts: Former client argued that his 6th Amendment right of assistance of counsel was violated when the attorney refused to cooperate with the defendant in presenting perjured testimony. Holding: The Supreme Court held that the right to counsel does not extend to perjury. An attorney is required by ethical rules not to present false testimony. A counsels duty of loyalty is limited to legitimate and lawful conduct. Prof. Resp. of the Criminal Defense Lawyer: The Three Hardest Questions (i) Is it proper to cross-examine for the purpose of discrediting the reliability or creditability of an adverse witness who you know to be telling the truth? As long as you say nothing false, it is permitted by the model rules. (ii) Is it proper to put a witness on the stand when you know he will commit perjury? Not permitted to put forth false testimony (see MR 3.3). (iii) Is it proper to give legal advice when you have reason to believe that the knowledge you give him will tempt him to commit perjury? You can give legal advice as to the legal consequences of the proposed conduct. The Search for the Truth: An Umpireal View (Frankel) Adversary system rates truth too low among its values. The search for the truth fails too much of the time. Adversary system puts the loyalty with the client and not on the truth. Judges should play a larger role in the quest for the truth. Lawyers should be better trained in questioning and ethics to hunt for the truth. Solutions: (i) Modify the adversary ideal; (ii) make truth the paramount objective; (iii) Impose upon the parties a duty to pursue the truth; (iv) change the ethical rules so that an attorney must tell the tribunal when any false statement by a client or witness has been made and allow questioning designed to arrive at the truth. A Causerie on Lawyers Ethics in Negotiation (Rubin) Nowhere is it ordained that the lawyer owes any general duty of candor or fairness to members of the bar or to laymen he may deal with as a negotiator. Unfortunately, practitioners behave as if anything not sanctioned by the rules is ok, but walking the line is not ethical behavior. Lawyers are more than protectors of the their clients; they are also keepers of a profession. Lawyers enjoy a monopoly on the practice of law and for that lawyers should look out for societys interests in achieving just resolutions to disputes.

21

A lawyer should act honestly and in good faith. A lawyer should not accept a result that is unconscionably unfair to the other party.

B. MASS. RULE 3.3(E) CANDOR TOWARD THE TRIBUNAL In a criminal case, defense counsel who knows that the defendant, the client, intends to testify falsely may not aid the client in constructing false testimony, and has a duty strongly to discourage the client from testifying falsely, advising that such a course is unlawful, will have substantial adverse consequences, and should not be followed. If a lawyer discovers this intention before accepting the representation of the client, the lawyer shall not accept the representation; if the lawyer discovers this intention before trial, the lawyer shall seek to withdraw from the representation, requesting any required permission. Disclosure of privileged or prejudicial information shall be made only to the extent necessary to effect the withdrawal. If disclosure of privileged or prejudicial information is necessary, the lawyer shall make an application to withdraw ex parte to a judge other than the judge who will preside at the trial and shall seek to be heard in camera and have the record of the proceeding, except for an order granting leave to withdraw, impounded. If the lawyer is unable to obtain the required permission to withdraw, the lawyer may not prevent the client from testifying. If a criminal trial has commenced and the lawyer discovers that the client intends to testify falsely at trial, the lawyer need not file a motion to withdraw from the case if the lawyer reasonably believes that seeking to withdraw will prejudice the client. If, during the client's testimony or after the client has testified, the lawyer knows that the client has testified falsely, the lawyer shall call upon the client to rectify the false testimony and, if the client refuses or is unable to do so, the lawyer shall not reveal the false testimony to the tribunal. In no event may the lawyer examine the client in such a manner as to elicit any testimony from the client the lawyer knows to be false, and the lawyer shall not argue the probative value of the false testimony in closing argument or in any other proceedings, including appeals. Rule 3.3(a) is intended to guide the conduct of the lawyer as an officer of the court as a prophylactic measure to protect against the contamination of the judicial process. Thus, for example, a lawyer who knows that a client has committed fraud on a tribunal and has refused to rectify it must disclose that fraud to avoid assisting the client's fraudulent act. Commonwealth v. Curtis Mitchell Facts: Client appealed his conviction on the ground that he was denied constitutionally effective counsel when his trial counsel, relying on Mass. Rule 3.3(e), advised the judge at trial that the client would testify and present false testimony to the jury. After receiving instructions from the judge, trial counsel presented the clients testimony in narrative form and made a closing argument that reflected his understanding of his ethical obligations. Holding: The court held that counsel acted properly when he informed the judge of his intention to invoke Mass. Rule 3.3(e) since he did not disclose what expected testimony would be perjurious. Moreover, the court held that the narrative form continues to be a commonly accepted method of dealing with client perjury. C. MODEL RULE 4.1 TRUTHFULNESS IN STATEMENTS TO OTHERS In the course of representing a client a lawyer shall not knowingly: (a) Make a false statement of material fact or law to a third person; or (b) Fail to disclose a material fact when disclosure is necessary to avoid assisting a criminal or fraudulent act by a client, unless disclosure is prohibited by Rule 1.6. A lawyer is required to be truthful when dealing with others on a client's behalf, but generally has no affirmative duty to inform an opposing party of relevant facts. Under generally accepted conventions in negotiation, certain types of statements ordinarily are not taken as statements of material fact. Estimates of price or value placed on the subject of a transaction and a party's intentions as to an acceptable settlement of a claim are ordinarily in this category, and so is the existence of an undisclosed principal except where nondisclosure of the principal would constitute fraud.

22

In Re Beiny Facts: The beneficiary's counsel (Sullivan & Cromwell) sought documents from the liquidator of a defunct law firm, which represented the trustee, the beneficiary's sister. The documents bore directly on the issue of ownership of art objects allegedly improperly diverted by the trustee from the inventory of an antique company, in which the beneficiary held a 45 percent interest, to trusts situated in Liechtenstein over which the trustee had exclusive control. The beneficiary's counsel deliberately violated the rules governing the conduct of discovery in order to obtain, covertly and without opposition, sweeping disclosure from the liquidator of documents that were immune from disclosure by reason of the attorney-client privilege. Holding: The court disqualified the beneficiary's counsel citing its intentional misconduct and the resulting prejudice to the trustee. Cincinnati Bar Association v. Statzer Facts: During a deposition, an attorney conspicuously placed audiocassette tapes in front of her former legal assistant. By suggestively labeling the tapes and referring to them during questioning, the attorney implied that she had recorded conversations with the legal assistant that could impeach and personally embarrass the legal assistant. Holding: The court held that code of professional responsibilitys definition of tribunal as all courts and all other adjudicatory bodies should be read broadly to include depositions. Although depositions are conventionally conducted without direct judicial supervision, such proceedings are nevertheless always subject to judicial intervention and oversight. Nonetheless, the court held that certain mitigating factors (i.e., attorney had otherwise clean record) required tempering the sanction. Dissent: The dissent argued that attorneys must be held to higher ethical standards. Attorneys are not justified in employing deception even when they believe that the person with whom they are dealing with is being untruthful. To condone such conduct, the practice of law would quickly slide to the lowest levels of ethical behavior. Shafer v. Berger Facts: Homeowners who obtained an arbitration judgment against an insured home remodeler for fraud filed an action against the remodeler's insurer, and against a law firm and an individual attorney retained by the insurer to provide coverage advice in the lawsuit against the insured. Plaintiffs' complaint alleged that the individual attorney made a fraudulent statement about coverage to plaintiffs in letters to plaintiffs' attorney pointing to the "occurrence" provision in the policy and the statutory exclusion for willful acts to justify the insurer's payment of only $ 120,000 toward a judgment totaling three times as much. Plaintiffs alleged that the insurer had agreed with the insured to cover willful acts so as to avoid providing the insured with independent counsel. Holding: The Court of Appeal held that plaintiffs sufficiently alleged that the attorney made a fraudulent statement about coverage to plaintiffs, which they viewed as a statement of fact, along with justifiable reliance, to withstand a demurrer. The court also held that the litigation privilege did not shield defendant from liability for fraud because his alleged misrepresentations were made to a party standing in the shoes of an insured, and the application of the litigation privilege would be inconsistent with the purpose of Ins. Code, 11580, a more specific statute, which makes the judgment creditor of the insured a third party beneficiary of the insurance contract between the insurer and the insured. Section 1714.10, by its own terms, does not apply to cause of action against an attorney for a civil conspiracy with his or her client, where, as here, the attorney had an independent legal duty to the plaintiff. An attorney can be liable for the intentional tort of fraud, under California law, if there is: (i) a misrepresentation; (ii) knowledge of falsity; (iii) intent to defraud (i.e., to induce reliance); (iv) justifiable reliance; and (v) resulting damage. District of Columbia Ethics Opinion 323 Misrepresentations by an Attorney Employed by a Government Agency as Part of Official Duties Lawyers employed by government agencies who act in a non-representational official capacity in a manner they reasonably believe to be authorized by law do not violate Rule 8.4, if in the course of their employment, they make misrepresentations that are reasonably intended to further the conduct of their official duties.

23

Argued that Rule 8.4 applies only to conduct that calls into question a lawyers suitability to practice law it does not encompass all acts of deceit. Pointed to commentary to Rule 4.2s language, which stated that the authorized by law proviso is intended to permit government conduct that is valid under the [laws of the United States or the District of Columbia]. Argued that the rules of professional conduct are rules of reason. They should be interpreted with reference to the purpose of legal representation and of the law itself.

D. MODEL RULE 4.3 DEALING WITH UNREPRESENTED PERSONS In dealing on behalf of a client with a person who is not represented by counsel, a lawyer shall not state or imply that the lawyer is disinterested. When the lawyer knows or reasonably should know that the unrepresented person misunderstands the lawyer's role in the matter, the lawyer shall make reasonable efforts to correct the misunderstanding. The lawyer shall not give legal advice to an unrepresented person, other than the advice to secure counsel, if the lawyer knows or reasonably should know that the interests of such a person are or have a reasonable possibility of being in conflict with the interests of the client. In order to avoid a misunderstanding, a lawyer will typically need to identify the lawyers client and, where necessary, explain that the client has interests opposed to those of the unrepresented person. MR 4.3 does not prohibit a lawyer from negotiating the terms of a transaction or settling a dispute with an unrepresented person. VI. Communications and Difficult Clients A. MODEL RULE 1.4 COMMUNICATIONS (a) A lawyer shall: (1) Promptly inform the client of any decision or circumstance with respect to which the client's informed consent, as defined in Rule 1.0(e), is required by these Rules; (2) Reasonably consult with the client about the means by which the client's objectives are to be accomplished; (3) Keep the client reasonably informed about the status of the matter; (4) Promptly comply with reasonable requests for information; and (5) Consult with the client about any relevant limitation on the lawyer's conduct when the lawyer knows that the client expects assistance not permitted by the Rules of Professional Conduct or other law. (b) A lawyer shall explain a matter to the extent reasonably necessary to permit the client to make informed decisions regarding the representation. The guiding principle is that the lawyer should fulfill reasonable client expectations for information consistent with the duty to act in the client's best interests, and the client's overall requirements as to the character of representation. In some circumstances, a lawyer may be justified in delaying transmission of information when the client would be likely to react imprudently to an immediate communication. Thus, a lawyer might withhold a psychiatric diagnosis of a client when the examining psychiatrist indicates that disclosure would harm the client. A lawyer may not withhold information to serve the lawyer's own interest or convenience or the interests or convenience of another person. B. MODEL RULE 1.13 ORGANIZATION AS CLIENT (a) A lawyer employed or retained by an organization represents the organization acting through its duly authorized constituents. (b) If a lawyer for an organization knows that an officer, employee or other person associated with the organization is engaged in action, intends to act or refuses to act in a matter related to the representation that is a violation of a legal obligation to the organization, or a violation of law that reasonably might be imputed to the organization, and that is likely to result in substantial injury to the organization, then the lawyer shall proceed as is reasonably

24

necessary in the best interest of the organization. Unless the lawyer reasonably believes that it is not necessary in the best interest of the organization to do so, the lawyer shall refer the matter to higher authority in the organization, including, if warranted by the circumstances, to the highest authority that can act on behalf of the organization as determined by applicable law. (c) Except as provided in paragraph (d), if (1) Despite the lawyer's efforts in accordance with paragraph (b) the highest authority that can act on behalf of the organization insists upon or fails to address in a timely and appropriate manner an action or a refusal to act, that is clearly a violation of law, and (2) The lawyer reasonably believes that the violation is reasonably certain to result in substantial injury to the organization, then the lawyer may reveal information relating to the representation whether or not Rule 1.6 permits such disclosure, but only if and to the extent the lawyer reasonably believes necessary to prevent substantial injury to the organization. (d) Paragraph (c) shall not apply with respect to information relating to a lawyer's representation of an organization to investigate an alleged violation of law, or to defend the organization or an officer, employee or other constituent associated with the organization against a claim arising out of an alleged violation of law. (e) A lawyer who reasonably believes that he or she has been discharged because of the lawyer's actions taken pursuant to paragraphs (b) or (c), or who withdraws under circumstances that require or permit the lawyer to take action under either of those paragraphs, shall proceed as the lawyer reasonably believes necessary to assure that the organization's highest authority is informed of the lawyer's discharge or withdrawal. (f) In dealing with an organization's directors, officers, employees, members, shareholders or other constituents, a lawyer shall explain the identity of the client [the corporation] when the lawyer knows or reasonably should know that the organization's interests are adverse to those of the constituents with whom the lawyer is dealing. (g) A lawyer representing an organization may also represent any of its directors, officers, employees, members, shareholders or other constituents, subject to the provisions of Rule 1.7. If the organization's consent to the dual representation is required by Rule 1.7, the consent shall be given by an appropriate official of the organization other than the individual who is to be represented, or by the shareholders.

When one of the constituents of an organizational client communicates with the organization's lawyer in that person's organizational capacity, the communication is protected by Rule 1.6. Thus, by way of example, if an organizational client requests its lawyer to investigate allegations of wrongdoing, interviews made in the course of that investigation between the lawyer and the client's employees or other constituents are covered by Rule 1.6. This does not mean, however, that constituents of an organizational client are the clients of the lawyer. The lawyer may not disclose to such constituents information relating to the representation except for disclosures explicitly or impliedly authorized by the organizational client in order to carry out the representation or as otherwise permitted by Rule 1.6. The authority and responsibility provided in this Rule are concurrent with the authority and responsibility provided in other Rules. In particular, this Rule does not limit or expand the lawyer's responsibility under Rules 1.8, 1.16, 3.3 or 4.1. Paragraph (c) of this Rule supplements Rule 1.6(b) by providing an additional basis upon which the lawyer may reveal information relating to the representation, but does not modify, restrict, or limit the provisions of Rule 1.6(b)(1) - (6). When the client is a governmental organization, a different balance may be appropriate between maintaining confidentiality and assuring that the wrongful act is prevented or rectified, for public business is involved. Seems to imply more of an obligation to disclose and more leeway to do such. In Re Lindsey Facts: Independent Counsel (K. Starr) moved to compel testimony from Bruce Lindsey, White House Counsel and Asst. to the President, regarding information about possible criminal conduct in connection with Monica Lewinsky. Lindsey invoked government attorney-client privilege, executive

25

privilege and personal attorney-client privilege. Lindsey argued that attorney-client privilege permits a government lawyer to avoid a grand jury inquiry relating to the commission of possible crimes by government officials. Holding: The court determined that although a government attorney-client privilege exists in civil litigation, it should not be extended to the grand jury context because (i) government lawyers, unlike private lawyers, have a duty to protect the public interest, and invoking privilege to shield criminal behavior by officials is inconsistent with that duty and (ii) 28 USC 535(b), which requires federal employees to report criminal violations to the Attorney General, evidences a congressional policy in favor of disclosing, not shielding, criminal conduct. C. SARBANES-OXLEY ACT STANDARDS OF PROFESSIONAL CONDUCT FOR ATTORNEYS Requires an attorney to report evidence of a material violation, determined according to an objective standard, up the ladder within the issuer to the chief legal counsel or the chief executive officer or the equivalent. Requires an attorney, if the chief legal counsel or the chief executive officer does not respond appropriately to the evidence, to report the evidence to the audit committee, another committee of independent directors, or the full board of directors. Allows an attorney, without the consent of an issue client, to reveal confidential information related to his or her representation to the extent the attorney reasonably believes necessary to (i) prevent the issue from committing a material violation likely to cause substantial injury to the financial interests or property of the issue or investors; (ii) to prevent the issue from committing an illegal act; or (iii) to rectify the consequences of a material violation or illegal act in which the attorneys services have been used. Termed the noisy withdrawal. 17 C.F.R. 205.2 Definitions (e) Evidence of a material violation means credible evidence, based upon which it would be unreasonable, under the circumstances, for a prudent and competent attorney not to conclude that it is reasonably likely that a material violation has occurred, is ongoing, or is about to occur. (i) Material violation means a material violation of an applicable U.S. federal or state securities law, a material breach of fiduciary duty arising under U.S. federal or state law, or a similar material violation of any U.S. federal or state law. 17 C.F.R. 205.3(b)(2) Issuer as Client Duty to Report Evidence Unless the chief legal officer (or the equivalent thereof) reasonably believes that no material violation has occurred, is ongoing, or is about to occur, he or she shall take all reasonable steps to cause the issuer to adopt an appropriate response, and shall advise the reporting attorney thereof. The Bottom Line Obligations for revealing client confidence under MR 1.13 or S-Ox are discretionary. Mandatory to at least go up the ladder to the highest authority. If it involves a tribunal, there may be other obligations under MR 3.3. Or under Mass. Rule 4.1 you may be compelled to disclose. ABA Against Mandatory Noisy Withdrawal Rules ABA argues that implementation of a mandatory rule will: (i) destroy the issuers trust in their attorneys by creating conflicts between the attorneys personal interest in avoiding potential discipline and the desire of management to operate the business; (ii) discourage companies from consulting counsel on sensitive issues, or cause companies to withhold necessary facts; (iii) hinder attorney flexibility in responding to compliance questions; and (iv) encourage premature withdrawal by counsel to avoid application of the reporting out rule.

26

D. MODEL RULE 1.14 CLIENT WITH DIMINISHED CAPACITY (a) When a client's capacity to make adequately considered decisions in connection with a representation is diminished, whether because of minority, mental impairment or for some other reason, the lawyer shall, as far as reasonably possible, maintain a normal client-lawyer relationship with the client. (b) When the lawyer reasonably believes that the client has diminished capacity, is at risk of substantial physical, financial or other harm unless action is taken and cannot adequately act in the client's own interest, the lawyer may take reasonably necessary protective action, including consulting with individuals or entities that have the ability to take action to protect the client and, in appropriate cases, seeking the appointment of a guardian ad litem, conservator or guardian. (c) Information relating to the representation of a client with diminished capacity is protected by Rule 1.6. When taking protective action pursuant to paragraph (b), the lawyer is impliedly authorized under Rule 1.6(a) to reveal information about the client, but only to the extent reasonably necessary to protect the client's interests. The client may wish to have family members or other persons participate in discussions with the lawyer. When necessary to assist in the representation, the presence of such persons generally does not affect the applicability of the attorney-client evidentiary privilege. E. MODEL RULE 1.16 DECLINING OR TERMINATING REPRESENTATION (a) Except as stated in paragraph (c), a lawyer shall not represent a client or, where representation has commenced, shall withdraw from the representation of a client if: (1) The representation will result in violation of the rules of professional conduct or other law; (2) The lawyer's physical or mental condition materially impairs the lawyer's ability to represent the client; or (3) The lawyer is discharged. (b) Except as stated in paragraph (c), a lawyer may withdraw from representing a client if: (1) Withdrawal can be accomplished without material adverse effect on the interests of the client; (2) The client persists in a course of action involving the lawyer's services that the lawyer reasonably believes is criminal or fraudulent; (3) The client has used the lawyer's services to perpetrate a crime or fraud; (4) The client insists upon taking action that the lawyer considers repugnant or with which the lawyer has a fundamental disagreement; (5) The client fails substantially to fulfill an obligation to the lawyer regarding the lawyer's services and has been given reasonable warning that the lawyer will withdraw unless the obligation is fulfilled; (6) The representation will result in an unreasonable financial burden on the lawyer or has been rendered unreasonably difficult by the client; or (7) Other good cause for withdrawal exists. (c) A lawyer must comply with applicable law requiring notice to or permission of a tribunal when terminating a representation. When ordered to do so by a tribunal, a lawyer shall continue representation notwithstanding good cause for terminating the representation. (d) Upon termination of representation, a lawyer shall take steps to the extent reasonably practicable to protect a client's interests, such as giving reasonable notice to the client, allowing time for employment of other counsel, surrendering papers and property to which the client is entitled and refunding any advance payment of fee or expense that has not been earned or incurred. However, the lawyer may retain papers relating to the client to the extent permitted by other law. F. MODEL RULE 2.4 LAWYER SERVING AS THIRD-PARTY NEUTRAL (a) A lawyer serves as a third-party neutral when the lawyer assists two or more persons who are not clients of the lawyer to reach a resolution of a dispute or other matter that has arisen

27

between them. Service as a third-party neutral may include service as an arbitrator, a mediator or in such other capacity as will enable the lawyer to assist the parties to resolve the matter. (b) A lawyer serving as a third-party neutral shall inform unrepresented parties that the lawyer is not representing them. When the lawyer knows or reasonably should know that a party does not understand the lawyer's role in the matter, the lawyer shall explain the difference between the lawyer's role as a third-party neutral and a lawyer's role as one who represents a client.

G. MODEL RULE 4.2 COMMUNICATION WITH PERSON REPRESENTED BY COUNSEL In representing a client, a lawyer shall not communicate about the subject of the representation with a person the lawyer knows to be represented by another lawyer in the matter, unless the lawyer has the consent of the other lawyer or is authorized to do so by law or a court order. The Rule applies even though the represented person initiates or consents to the communication. A lawyer must immediately terminate communication with a person if, after commencing communication, the lawyer learns that the person is one with whom communication is not permitted by this Rule. Messing, Rudavsky & Weliky, P.C. v. President & Fellows of Harvard College Facts: Sergeant employed by university police department brought claims for sex discrimination and retaliation against private university. The Commission Against Discrimination found that ex parte contacts by the law firm representing the sergeant, with two lieutenants, two patrol officers, and a dispatcher employed by the university, violated the rules of professional conduct. Holding: The Supreme Judicial Court held that the law firms ex parte contacts did not violate the rules of professional conduct. Reasoning: A lawyer is barred from making ex parte contacts with the employees of a corporation, under the rule of professional conduct prohibiting a lawyer from communicating with a represented party in the absence of that party's counsel, only as to: (i) employees who exercise managerial responsibility with regard to the subject of the litigation, (ii) employees who are alleged to have committed the wrongful acts at issue in the litigation, and (iii) employees who have the authority to make decisions about the course of the litigation or have managing authority sufficient to give them the right to speak for, and bind, the corporation. VII. Starting A Law Practice A. MODEL RULE 1.2 SCOPE OF REPRESENTATION & ALLOCATION OF AUTHORITY (a) Subject to paragraphs (c) and (d), a lawyer shall abide by a client's decisions concerning the objectives of representation and, as required by Rule 1.4, shall consult with the client as to the means by which they are to be pursued. A lawyer may take such action on behalf of the client as is impliedly authorized to carry out the representation. A lawyer shall abide by a client's decision whether to settle a matter. In a criminal case, the lawyer shall abide by the client's decision, after consultation with the lawyer, as to a plea to be entered, whether to waive jury trial and whether the client will testify. (b) A lawyer's representation of a client, including representation by appointment, does not constitute an endorsement of the client's political, economic, social or moral views or activities. (c) A lawyer may limit the scope of the representation if the limitation is reasonable under the circumstances and the client gives informed consent. (d) A lawyer shall not counsel a client to engage, or assist a client, in conduct that the lawyer knows is criminal or fraudulent, but a lawyer may discuss the legal consequences of any proposed course of conduct with a client and may counsel or assist a client to make a good faith effort to determine the validity, scope, meaning or application of the law. Legal representation should not be denied to people who are unable to afford legal services, or whose cause is controversial or the subject of popular disapproval. By the same token, representing a client does not constitute approval of the clients views or activities.

28

The fact that a client may use a lawyers advice in a course of action that is criminal or fraudulent of itself does not make a lawyer party to the course of action. There is a critical distinction between presenting an analysis of legal aspects of questionable conduct and recommending the means by which a crime or fraud might be committed with impunity. B. MODEL RULE 1.5 FEES (a) A lawyer shall not make an agreement for, charge, or collect an unreasonable fee or an unreasonable amount for expenses. The factors to be considered in determining the reasonableness of a fee include the following: (1) the time and labor required, the novelty and difficulty of the questions involved, and the skill requisite to perform the legal service properly; (2) the likelihood, if apparent to the client, that the acceptance of the particular employment will preclude other employment by the lawyer; (3) the fee customarily charged in the locality for similar legal services; (4) the amount involved and the results obtained; (5) the time limitations imposed by the client or by the circumstances; (6) the nature and length of the professional relationship with the client; (7) the experience, reputation, and ability of the lawyer or lawyers performing the services; and (8) whether the fee is fixed or contingent. (b) The scope of the representation and the basis or rate of the fee and expenses for which the client will be responsible shall be communicated to the client, preferably in writing, before or within a reasonable time after commencing the representation, except when the lawyer will charge a regularly represented client on the same basis or rate. Any changes in the basis or rate of the fee or expenses shall also be communicated to the client. (c) A fee may be contingent on the outcome of the matter for which the service is rendered, except in a matter in which a contingent fee is prohibited by paragraph (d) or other law. A contingent fee agreement shall be in a writing signed by the client and shall state the method by which the fee is to be determined, including the percentage or percentages that shall accrue to the lawyer in the event of settlement, trial or appeal; litigation and other expenses to be deducted from the recovery; and whether such expenses are to be deducted before or after the contingent fee is calculated. The agreement must clearly notify the client of any expenses for which the client will be liable whether or not the client is the prevailing party. Upon conclusion of a contingent fee matter, the lawyer shall provide the client with a written statement stating the outcome of the matter and, if there is a recovery, showing the remittance to the client and the method of its determination. (d) A lawyer shall not enter into an arrangement for, charge, or collect: (1) any fee in a domestic relations matter, the payment or amount of which is contingent upon the securing of a divorce or upon the amount of alimony or support, or property settlement in lieu thereof; or (2) a contingent fee for representing a defendant in a criminal case. (e) A division of a fee between lawyers who are not in the same firm may be made only if: (1) the division is in proportion to the services performed by each lawyer or, each lawyer assumes joint responsibility for the representation; (2) the client agrees to the arrangement, including the share each lawyer will receive, and the agreement is confirmed in writing; and (3) the total fee is reasonable. Goldfarb v. Virginia State Bar Facts: Petitioners, a husband and wife who contracted to purchase a home, contacted a lawyer to provide a title examination. The lawyer quoted the fee suggested in a minimum fee schedule published by respondent county bar association. When petitioners were unable to find a lawyer who would examine the title for less than the fee fixed by the schedule, petitioners commenced action against respondent state and county bar associations, alleging they violated the Sherman Act, 15 U.S.C. 1, by promulgating and enforcing a minimum fee schedule for lawyers. Petitioners challenged court of

29

appeals ruling reversing a district court decision in favor of petitioners, on grounds those respondents were immune from antitrust liability. Holding: The Supreme Court reversed the court of appeals decision because anticompetitive conduct by lawyers was within the reach of the Sherman Act. The court noted that the fact that the state bar association was a state agency for some limited purposes did not create an antitrust shield. Hence, the court ruled that respondents were liable since their activities involved blatant price-fixing, which restrained competition and harmed consumers. In the Matter of Fordham Facts: The Mass. Bar counsel filed a petition for discipline against the attorney alleging that the attorney charged excessive fees for a drunken driving case. Though an experienced litigator, Fordham had never tried a drunk driving case before and as a result of inexperience in the field, he charged $50,022.25, reflecting 227 hours of billed time for a case that would normally range between $3-10k. Holding: The court found that (1) it had jurisdiction to hear the appeal, (2) expert testimony indicated that the number of hours devoted to the case was substantially in excess of the hours a prudent experienced lawyer would have spent, (3) the attorney's inexperience did not justify the high fee, as the client should not have paid for the education of the attorney, and (4) the disciplinary rule regarding excessive fees did not inquire into the nature of the attorney's good faith or diligence. C. MODEL RULE 5.1 RESPONSIBILITIES OF PARTNERS, MANAGERS & SUPERVISORY LAWYERS (a) A partner in a law firm, and a lawyer who individually or together with other lawyers possesses comparable managerial authority in a law firm, shall make reasonable efforts to ensure that the firm has in effect measures giving reasonable assurance that all lawyers in the firm conform to the Rules of Professional Conduct. (b) A lawyer having direct supervisory authority over another lawyer shall make reasonable efforts to ensure that the other lawyer conforms to the Rules of Professional Conduct. (c) A lawyer shall be responsible for another lawyer's violation of the Rules of Professional Conduct if: (1) the lawyer orders or, with knowledge of the specific conduct, ratifies the conduct involved; or (2) the lawyer is a partner or has comparable managerial authority in the law firm in which the other lawyer practices, or has direct supervisory authority over the other lawyer, and knows of the conduct at a time when its consequences can be avoided or mitigated but fails to take reasonable remedial action. D. MODEL RULE 5.2 RESPONSIBILITIES OF A SUBORDINATE LAWYER (a) A lawyer is bound by the Rules of Professional Conduct notwithstanding that the lawyer acted at the direction of another person. (b) A subordinate lawyer does not violate the Rules of Professional Conduct if that lawyer acts in accordance with a supervisory lawyer's reasonable resolution of an arguable question of professional duty. E. MODEL RULE 5.4 PROFESSIONAL INDEPENDENCE OF A LAWYER (a) A lawyer or law firm shall not share legal fees with a non-lawyer, except that: (1) An agreement by a lawyer with the lawyer's firm, partner, or associate may provide for the payment of money, over a reasonable period of time after the lawyer's death, to the lawyer's estate or to one or more specified persons; (2) A lawyer who purchases the practice of a deceased, disabled, or disappeared lawyer may, pursuant to the provisions of Rule 1.17, pay to the estate or other representative of that lawyer the agreed-upon purchase price; (3) A lawyer or law firm may include non-lawyer employees in a compensation or retirement plan, even though the plan is based in whole or in part on a profit-sharing arrangement; and (4) A lawyer may share court-awarded legal fees with a nonprofit organization that employed, retained or recommended employment of the lawyer in the matter. (b) A lawyer shall not form a partnership with a non-lawyer if any of the activities of the partnership consist of the practice of law.

30

(c) A lawyer shall not permit a person who recommends, employs, or pays the lawyer to render legal services for another to direct or regulate the lawyer's professional judgment in rendering such legal services. (d) A lawyer shall not practice with or in the form of a professional corporation or association authorized to practice law for a profit, if: (1) A non-lawyer owns any interest therein, except that a fiduciary representative of the estate of a lawyer may hold the stock or interest of the lawyer for a reasonable time during administration; (2) A non-lawyer is a corporate director or officer thereof or occupies the position of similar responsibility in any form of association other than a corporation; or (3) A non-lawyer has the right to direct or control the professional judgment of a lawyer.

F.

MODEL RULE 7.1 COMMUNICATIONS CONCERNING A LAWYERS SERVICES A lawyer shall not make a false or misleading communication about the lawyer or the lawyer's services. A communication is false or misleading if it contains a material misrepresentation of fact or law, or omits a fact necessary to make the statement considered as a whole not materially misleading.

G. MODEL RULE 7.2 ADVERTISING (a) Subject to the requirements of Rules 7.1 and 7.3, a lawyer may advertise services through written, recorded or electronic communication, including public media. (b) A lawyer shall not give anything of value to a person for recommending the lawyer's services except that a lawyer may: (1) Pay reasonable costs of advertisements or communications permitted by this Rule; (2) Pay the usual charges of a legal service plan or a not-for-profit or qualified lawyer referral service. A qualified lawyer referral service is a lawyer referral service that has been approved by an appropriate regulatory authority; (3) Pay for a law practice in accordance with Rule 1.17; and (4) Refer clients to another lawyer or a non-lawyer professional pursuant to an agreement not otherwise prohibited under these Rules that provides for the other person to refer clients or customers to the lawyer, if: (i) The reciprocal referral agreement is not exclusive, and (ii) The client is informed of the existence and nature of the agreement. (c) Any communication made pursuant to this rule shall include the name and office address of at least one lawyer or law firm responsible for its content. Bates v. State Bar of Arizona Facts: Attorneys were licensed to practice law in the State of Arizona. In direct violation of a disciplinary rule prohibiting attorney advertising that had been promulgated by the State Bar of Arizona, the attorneys placed an advertisement in an Arizona newspaper, which stated that they were offering legal services at reasonable fees and which listed their fees for certain services. The attorneys sought review of the rule after it was recommended that appellants be temporarily suspended from the practice of law. The state supreme court rejected appellants' arguments that the disciplinary rule violated 15 U.S.C.S. 1 and 2 of the Sherman Act and infringed on their First Amendment rights. Holding: The Supreme Court affirmed the lower court's finding that the regulation was shielded from Sherman Act attack because the rule was an activity of the state acting as sovereign. However, that part of the judgment dealing with the First Amendment was reversed upon the Court's holding that advertising by attorneys could not be subjected to blanket suppression and because the truthful advertisement at issue was found to be protected by the First Amendment. H. MODEL RULE 7.3 DIRECT CONTACT WITH PROSPECTIVE CLIENTS (SOLICITATION) (a) A lawyer shall not by in-person, live telephone or real-time electronic contact solicit professional employment from a prospective client when a significant motive for the lawyer's doing so is the lawyer's pecuniary gain, unless the person contacted: (1) Is a lawyer; or (2) Has a family, close personal, or prior professional relationship with the lawyer.

31

(b) A lawyer shall not solicit professional employment from a prospective client by written, recorded or electronic communication or by in-person, telephone or real-time electronic contact even when not otherwise prohibited by paragraph (a), if: (1) The prospective client has made known to the lawyer a desire not to be solicited by the lawyer; or (2) The solicitation involves coercion, duress or harassment. (c) Every written, recorded or electronic communication from a lawyer soliciting professional employment from a prospective client known to be in need of legal services in a particular matter shall include the words ""Advertising Material" on the outside envelope, if any, and at the beginning and ending of any recorded or electronic communication, unless the recipient of the communication is a person specified in paragraphs (a)(1) or (a)(2). (d) Notwithstanding the prohibitions in paragraph (a), a lawyer may participate with a prepaid or group legal service plan operated by an organization not owned or directed by the lawyer that uses in-person or telephone contact to solicit memberships or subscriptions for the plan from persons who are not known to need legal services in a particular matter covered by the plan.

In Re Primus Facts: An attorney affiliated with the ACLU sent a letter to a woman who had been sterilized as a condition of continued receipt of medical assistance. The attorney informed the woman of the ACLU's offer of free legal representation, in the event that she ever wished to institute suit against the doctor who performed the sterilization procedure. The South Carolina supreme court affirmed an order finding that petitioner violated its disciplinary rules by attempting to solicit a client for a nonprofit organization which, as its primary purpose, rendered legal services. Holding: The Supreme Court reversed, holding that the lower court's application of its state disciplinary rules violated the First and Fourteenth Amendments. The court found that attorney's actions were undertaken to express personal political beliefs and to advance the civil liberties objectives of the ACLU, rather than to derive financial gain. The state's action in punishing petitioner for soliciting a prospective litigant by mail, on behalf of the ACLU, did not withstand the exacting scrutiny applicable to limitations on core First Amendment rights. Ohralik v. Ohio State Bar Association Facts: An attorney approached two accident victims, urged his services upon them, induced their affirmative responses to his solicitation while carrying a concealed tape recorder, and emphasized that his fee would come out of their recovery. The Ohio supreme court held that the attorney's conduct was not constitutionally protected, publicly reprimanded him, and indefinitely suspended him. Holding: The Supreme Court affirmed, holding that the state's application of disciplinary rules to petitioner's conduct did not offend the First Amendment. The Court found that the facts presented a striking example of the potential for overreaching that was inherent in lawyers' in-person solicitation of professional employment. The facts also demonstrated the need for prophylactic regulation in furtherance of states' interests in protecting the lay public. Reasoning: In addition to the general interest in protecting consumers and regulating commercial transactions, the state bears a special responsibility for maintaining standards among members of the licensed professions. Shapero v. Kentucky Bar Association Facts: Attorney applied to the Kentucky Attorneys Advertising Commission for approval of a letter that he proposed to send to potential clients who had a foreclosure suit filed against them. The Commission did not find the letter false or misleading. Nevertheless, it declined to approve petitioner's proposal on the ground that a then-existing Kentucky Supreme Court rule prohibited the mailing or delivery of written advertisements precipitated by a specific event or occurrence involving or relating to the addressee as distinct from the general public. Pursuing the Commission's suggestion, petitioner sought from the ethics committee of respondent Kentucky Bar Association an advisory opinion as to the rule's validity. The ethics committee upheld the rule. The Kentucky Supreme Court replaced the rule with ABA Model Code of Prof'l Responsibility 7.3, which prohibited targeted, direct-mail solicitation by lawyers for pecuniary gain, without a particularized finding that the solicitation is false or misleading.

32

Holding: The Supreme Court reversed. Although unable to agree on an opinion regarding whether the attorney's proposed letter was particularly overreaching and therefore unworthy of First Amendment protection, six members of the court agreed that a blanket prohibition of the kind contained in Rule 7.3 is inconsistent with the First Amendment. Reasoning: A state may not, consistent with the First Amendment, as applied to the states through the Fourteenth Amendment, categorically prohibit lawyers from soliciting legal business for pecuniary gain by sending truthful and non-deceptive letters to potential clients known to face particular legal problems. Dissent: O'Connor, J., joined by Rehnquist, Ch. J., and Scalia, J., dissented, expressing the view that (i) the states should have considerable latitude to ban advertising that is potentially or demonstrably misleading, as well as truthful advertising that undermines the substantial governmental interest in promoting the high ethical standards that are necessary in the legal profession, (ii) Rule 7.3 is no broader than is necessary to advance a substantial governmental interest, and (iii) the proposed letter could permissibly be banned under Rule 7.3. Florida Bar v. Went For It, Inc. Facts: Two rules of the Florida Bar, taken together, forbade lawyers to send, directly or indirectly, targeted direct-mail solicitations of business to victims of an accident or disaster or to relatives of such victims during a 30-day period after the accident or disaster. The rules were adopted as a result of a 2year study by the Florida Bar of the effects of lawyer advertising on public opinion. A Florida lawyer and his wholly owned lawyer referral service, challenging the rules, filed an action for declaratory and injunctive relief against the Florida Bar. The District Court disagreed with the study, entered summary judgment for the lawyers, and expressed the view that the rules violated the Constitution's First, Fifth, and Fourteenth Amendments, as the rules (a) were not justified by a government interest in preventing undue influence, intimidation, and overreaching by lawyers, (b) were not narrowly tailored in a manner that advanced the asserted government interest, (c) did not constitute a valid time, place, and manner restriction, and (d) substantially impaired the availability of truthful and relevant information as to legal services. The Eleventh Circuit, in affirming, expressed the view that the rules violated the First Amendment, as the rules could not be justified by an asserted government interest in protecting the privacy of recent injury victims or their family members. Holding: The Supreme Court reversed, holding that the 30-day restriction withstood scrutiny under the test for determining the validity, under the First Amendment's free speech guarantee, of a restriction on commercial speech, as (1) the Florida Bar had asserted a substantial state interest in (a) protecting the privacy and tranquility of potential clients from commercial intrusion upon their personal grief in times of trauma, and (b) preventing the outrage and irritation with the statelicensed legal profession that the practice of direct solicitation only days after accidents had engendered; (2) evidence adduced by the bar was sufficient to establish that the restriction targeted a concrete, non-speculative harm; and (3) the restriction was reasonably tailored to its stated objective. Dissent: Kennedy, J., joined by Stevens, Souter, and Ginsburg, JJ., dissenting, expressed the view that (1) restrictions on speech are not justified on the ground that the expression might offend the listener; (2) direct solicitation by attorneys might serve vital purposes and promote the administration of justice; (3) the evidence offered in support of the 30-day restriction fell short of demonstrating that the asserted harms were real; (4) the restriction prohibited far more speech than necessary to serve the purported state interest; and (5) the Constitution did not permit the state to promote the public image of the legal profession by suppressing information about the profession's business aspects. I. MODEL RULE 7.4 COMMUNICATION OF FIELDS OF PRACTICE AND SPECIALIZATION (a) A lawyer may communicate the fact that the lawyer does or does not practice in particular fields of law. (b) A lawyer admitted to engage in patent practice before the U.S. Patent and Trademark Office may use the designation "Patent Attorney" or a substantially similar designation. (c) A lawyer engaged in Admiralty practice may use the designation ""Admiralty," "Proctor in Admiralty" or a substantially similar designation. (d) A lawyer shall not state or imply that a lawyer is certified as a specialist in a particular field of law, unless:

33

(1) The lawyer has been certified as a specialist by an organization that has been approved by an appropriate state authority or that has been accredited by the American Bar Association; and (2) The name of the certifying organization is clearly identified in the communication.

In Mass., if a lawyer communicates that he is a specialist or an expert in a particular field of law, he or she will be held to the higher expert standard in a malpractice lawsuit. J. MODEL RULE 7.5 FIRM NAMES AND LETTERHEADS (a) A lawyer shall not use a firm name, letterhead or other professional designation that violates Rule 7.1. A trade name may be used by a lawyer in private practice if it does not imply a connection with a government agency or with a public or charitable legal services organization and is not otherwise in violation of Rule 7.1. (b) A law firm with offices in more than one jurisdiction may use the same name or other professional designation in each jurisdiction, but identification of the lawyers in an office of the firm shall indicate the jurisdictional limitations on those not licensed to practice in the jurisdiction where the office is located. (c) The name of a lawyer holding a public office shall not be used in the name of a law firm, or in communications on its behalf, during any substantial period in which the lawyer is not actively and regularly practicing with the firm. (d) Lawyers may state or imply that they practice in a partnership or other organization only when that is the fact. If a private firm uses a trade name that includes a geographical name such as Springfield Legal Clinic, an express disclaimer that it is a public legal aid agency may be required to avoid a misleading implication. Any firm name that includes the name of a deceased partner is, strictly speaking, a trade name. It is misleading to use the name of a lawyer not associated with the firm, or a predecessor of the firm, or the name of a non-lawyer. VIII. Bar Admissions A. THE GATEKEEPERS AND YOU Bar admissions are controlled in three ways: Must graduate from a 3 year, ABA accredited law school (narrow exceptions apply). Must pass the bar exam. Must be of good character. In determining good character, an applicants answer that presents a concern does not inevitably preclude individual from admission. Candidate will be referred to a Character and Fitness Committee to seek further explanation. B. MODEL RULE 8.1 BAR ADMISSION AND DISCIPLINARY MATTERS An applicant for admission to the bar, or a lawyer in connection with a bar admission application or in connection with a disciplinary matter, shall not: (a) Knowingly make a false statement of material fact; or (b) Fail to disclose a fact necessary to correct a misapprehension known by the person to have arisen in the matter, or knowingly fail to respond to a lawful demand for information from an admissions or disciplinary authority, except that this rule does not require disclosure of information otherwise protected by Rule 1.6. The duty imposed by this Rule extends to persons seeking admission to the bar as well as to lawyers. Hence, if a person makes a material false statement in connection with an application for admission, it may be the basis for subsequent disciplinary action if the person is admitted, and in any event may be relevant in a subsequent admission application.

34

Application of Gahan Facts: Gahan sought review of a determination by respondent Minnesota State Board of Law Examiners that the applicant did not meet the standards required of applicants for admission to practice law in Minnesota. The Board found that the Gahan lacked recognition and appreciation of the underlying moral and social obligation that arose when he was entrusted with his student loans. Holding: The court held that although there was nothing connected with the applicant's bankruptcy to suggest that there was any fraud, deceit, or conduct that could be considered moral turpitude, the applicant was properly denied admission on the grounds of insufficient moral character. Reasoning: Although Gahan was protected by federal rights for those who elected to file voluntary bankruptcy, his conduct prior to the bankruptcy surrounding his financial responsibility and his default on the student loans was properly considered to judge his moral character. Applicants who flagrantly disregarded the rights of others and defaulted on serious financial obligations, such as student loans, were lacking in good moral character if the default was neglectful, irresponsible, and could not be excused by a compelling hardship that was reasonably beyond his/her control. In Re Applicant Facts: An applicant for admission to the Florida Bar refused to complete item 28(b) on the application, which asked if applicant had ever received regular treatment for amnesia, insanity, emotional disturbance, or a nervous or mental disorder. When the Florida Board of Bar Examiners (board) refused to complete his application with the altered release that applicant submitted regarding item 28(b), applicant sought review of the decision. Applicant argued that the question violated his constitutional right to privacy as guaranteed by the Florida State Constitution. The board argued that the state had a compelling interest in the information given the constant contact an attorney has with the public, as well as the pressures inherent within the practice that any attorney must be mentally and emotionally fit to handle. Holding: The court agreed with the board and upheld the requirement that applicant complete item 28(b) and file an unaltered completed release. The court held that the board had employed the least intrusive means to achieving its compelling state interest and affirmed the board's decision to refuse to complete applicant's bar application. Reasoning: It is imperative for the protection of the public that applicants to the Florida Bar be thoroughly screened by the Board of Bar Examiners (board). Necessarily, the board must ask questions in this screening process which are of a personal nature and which would not otherwise be asked of persons not applying for a position of public trust and responsibility. Because of a lawyer's constant interaction with the public, a wide range of factors must be considered which would not customarily be considered in the licensing of tradesmen and businessmen. The inquiry into an applicant's past history of regular treatment for emotional disturbance or nervous or mental disorder furthers a legitimate state interest since mental fitness and emotional stability are essential to the ability to practice law in a manner not injurious to the public. In Re Hale Facts: Committee on Character and Fitness denied Hales application for admission on the basis of his open advocacy of racially obnoxious beliefs. The Hearing Panel found that his publically displayed views were diametrically opposed to the letter and spirit of the Rules of Professional Conduct. The Inquiry Panel held that in regulating the conduct of attorneys, certain fundamental truths of equality and non-discrimination must be preferred over the values found in the First Amendment. Holding: The Supreme Court denied the writ of certiorari, letting the Illinois court decision stand. Dissent: The Committee seems to hold that it may deny petitioner's application for admission to the bar without finding that petitioner has engaged in any specific conduct that would have violated a disciplinary rule if petitioner were already a lawyer. The Committee merely speculates that petitioner is on a collision course with the Rules of Professional Conduct" and that, if admitted, he will in the future "find himself before the Attorney Registration and Disciplinary Commission." I believe this court should address whether it is appropriate for the Committee to base its assessment of an applicant's character and fitness on speculative predictions of future actionable misconduct.

35

In Re Alger Hiss Facts: Alger Hiss was struck from the roll of Massachusetts lawyers on August 1, 1952. Hiss was convicted of two counts of perjury in his testimony before a federal grand jury. In particular, the attorney was found to have testified falsely that he had never, nor had his wife in his presence, turned over documents or copies of documents of the United States Department of State or of any other organization of the federal government to a person or to any other unauthorized person and that he thought he could have said definitely that he had not seen the person after a certain date. After the he had exhausted his rights of appeal he served three and one-half years incarceration. In 1975, Hiss brought a petition for reinstatement as an attorney. Holding: The court granted the attorney's petition for reinstatement. The court held that the attorney was trustworthy even though he was unwilling to admit his past mistakes. Reasoning: Conviction of perjury of a member of the bar of this Commonwealth and his subsequent disbarment were "conclusive evidence of his lack of moral character at the time of his removal from office." However, the court could not say that any offense is so grave that an attorney disbarred therefore is automatically precluded from subsequently attempting to demonstrate that he has achieved a "present fitness," and has led a sufficiently exemplary life, to be reinstated as an attorney. Statements of guilt and repentance may be desirable as evidence that the disbarred attorney recognizes his past wrongdoing and will attempt to avoid repetition in the future. However, to satisfy the requirements of present good moral character in the tests for reinstatement noted above, it is sufficient that petitioner adduce substantial proof that he has such an appreciation of the distinctions between right and wrong in the conduct of men toward each other as will make him a fit and safe person to engage in the practice of law. Such an appreciation, if deeply felt and strongly anchored, will serve as a firm foundation and justification for the order of reinstatement. Mere words of repentance are easily uttered and just as easily forgotten. In judging whether a petitioner satisfies these standards and is demonstrated the requisite rehabilitation since disbarment, it is necessary to look to (i) the nature of the original offense for which petitioner is disbarred, (ii) petitioner's character, maturity, and experience at the time of his disbarment, (iii) petitioner's occupations and conduct in the time since his disbarment, (iv) the time elapsed since the disbarment, and (v) petitioner's present competence in legal skills. In Re Hinson-Lyles Facts: The applicant, in her application to sit for the July 2002 Louisiana bar examination, disclosed that she was convicted of a felony sexual offense in 1999. She was a teacher, 23-years-old, and had engaged in a sexual relationship with a male student, who was 14 at the time of the sexual conduct. She drank alcohol with him and allowed him to smoke marijuana that he had brought to her home. The court had granted the applicant permission to sit for the bar, subject to he condition that upon her successful completion of the exam, she apply to the court for the appointment of a commissioner to take character and fitness evidence. She then passed the bar exam and at the conclusion of the hearing, the commissioner issued detailed findings of fact and conclusions of law and recommended that petitioner be conditionally admitted to the practice of law in Louisiana, subject to a probationary period of two years. Holding: Despite the commissioners findings, the court denied her petition seeking admission to the Bar of the State of Louisiana, stating that serious character and fitness concerns were present. Reasoning: The Supreme Court of Louisiana has the exclusive and plenary power to define and regulate all facets of the practice of law, including the admission of attorneys to the Bar of the State. Among other requirements for admission to the Bar, applicants must demonstrate by competent evidence that they have "good moral character and the fitness necessary to practice law in the State of Louisiana. The primary purpose of character and fitness screening is to assure the protection of the public and to safeguard the administration of justice. The term "good moral character" includes, but is not limited to, the qualities of honesty, fairness, candor, trustworthiness, observances of fiduciary responsibility and of the laws of the State of Louisiana and of the United States of America, and a respect for the rights of other persons. Dissent: The dissent argued that Court needed to develop definitive rules as to what conduct may be rehabilitated and what conduct is a bar to admission both would give useful guidance to applicants and would help avoid wasting time and valuable resources.

36

IX. Bar Discipline and Malpractice A. THE DISCIPLINARY PROCESS (IN MASS.) 1st. Anyone can initiate a Mass. Board of Bar Overseers complaint by filing a BBO-1 form. 2nd. The Bar Counsels office will then review the complaint and will seek a reply from the lawyer. 3rd. The Bar Counsel can then close the matter, or recommend an admonition, public reprimand or prosecution of formal charges. 4th. The Bar Counsels recommendations then go to a Reviewing Board Member and if that member believes that the charges would warrant public discipline if proved by a preponderance of the evidence, a Hearing Committee is established. Committee is subject to review by the Board and, where appropriate, the S.J.C. itself. The Board of Bar Overseers has a wide range of possible sanctions, ranging from a private letter of reprimand that is just a gentle reminder, requiring a few review classes in professional ethics, suspensions or disbarment for life. These are not criminal proceedings and there is a fundamental question of whether being a lawyer is a right or a privilege. B. MODEL RULE 8.3 REPORTING PROFESSIONAL MISCONDUCT (SNITCH RULE) (a) A lawyer who knows that another lawyer has committed a violation of the Rules of Professional Conduct that raises a substantial question as to that lawyer's honesty, trustworthiness or fitness as a lawyer in other respects, shall inform the appropriate professional authority. (b) A lawyer who knows that a judge has committed a violation of applicable rules of judicial conduct that raises a substantial question as to the judge's fitness for office shall inform the appropriate authority. (c) This Rule does not require disclosure of information otherwise protected by Rule 1.6 or information gained by a lawyer or judge while participating in an approved lawyers assistance program. If a lawyer were obliged to report every violation of the Rules, the failure to report any violation would itself be a professional offense. This Rule limits the reporting obligation to those offenses that a self-regulating profession must vigorously endeavor to prevent. A measure of judgment is, therefore, required in complying with the provisions of this Rule. The term substantial refers to the seriousness of the possible offense and not the quantum of evidence of which the lawyer is aware. C. MODEL RULE 8.4 MISCONDUCT It is professional misconduct for a lawyer to: (a) Violate or attempt to violate the Rules of Professional Conduct, knowingly assist or induce another to do so, or do so through the acts of another; (b) Commit a criminal act that reflects adversely on the lawyer's honesty, trustworthiness or fitness as a lawyer in other respects; (c) Engage in conduct involving dishonesty, fraud, deceit or misrepresentation; (d) Engage in conduct that is prejudicial to the administration of justice; (e) State or imply an ability to influence improperly a government agency or official or to achieve results by means that violate the Rules of Professional Conduct or other law; (f) Knowingly assist a judge or judicial officer in conduct that is a violation of applicable rules of judicial conduct or other law. Attorneys must report any [of their own] criminal violations to their bar associations each year. Acts outside the jurisdiction or the country are considered criminal only if they violate the laws of the state to which you are admitted to the bar (i.e., if it is illegal in Massachusetts, it is illegal regardless of where it takes place). Standard of proof for convictions under MR 8.4 is a preponderance of evidence. Note that Subsection (c) does not require a criminal act.

37

Subjection (d) applies to a lawyer who, in the course of representing a client, knowingly manifests by words or conduct, bias or prejudice based upon race, sex, religion, national origin, disability, age, sexual orientation or socioeconomic status, violates paragraph (d) when such actions are prejudicial to the administration of justice. MR 8.4 is very broad and the question is how broad can it be before it fails to provide notice. In Re Ruffalo Facts: Ruffalo was ordered suspended from the practice of law by the state supreme court on two findings of alleged misconduct. The charges were related to the attorney's hiring of an investigator to gather information relating to railroads. During testimony it was discovered that the investigator had also gathered and provided information on cases involving his employer. Following this an additional charge was added. Following the state proceedings, the federal district court, in considering whether the attorney should be barred from practice before the district court, found that there was no misconduct. However, the court of appeals disbarred him from practice in the court of appeals. Holding: The Supreme Court reversed the judgment of the Court of Appeals. The court held that the attorney was deprived of procedural due process because the charges against him were improperly amended on the basis of his testimony in the proceedings. The charge had to be known before the proceedings commenced. Reasoning: When proceedings for disbarment are not taken for matters occurring in open court, in the presence of the judges, notice should be given to the attorney of the charges made and opportunity afforded him for explanation and defense. Therefore, one of the conditions the court considers in determining whether disbarment by a state should be followed by disbarment in the federal courts is whether the state procedure from want of notice or opportunity to be heard was wanting in due process. Bar argues that employment arrangement between Ruffalo and Orlando encouraged Orlando to be disloyal to his company. Bar also argues that Ruffalo inappropriately requested Orlando to steer potential clients. Counsel points out that Ruffalo did not ask Orlando to solicit clients, did not have Orlando work for him during his work hours and did not have Orlando investigate accident that occurred in his companys rail yard. Attorney Grievance Commission of Maryland v. Sheinbein Facts: Attorney discovered that his teen-aged son had committed a murder, acted to hold off the police investigation, and arranged for his son to fly to Israel. The attorney testified to a grand jury that he acted in the belief that his son had acted in self-defense and to prevent his son from committing suicide, but by the time of the disciplinary proceedings the father had also fled to Israel. Holding: The court found no clear error in the hearing judge's determination that the attorney had violated Md. R. Prof. Conduct 8.4(b), (d) by engaging in conduct indicating his unfitness to practice and by obstructing justice. The attorney had committed all the elements of common law obstructing or hindering a police officer and would have been arrested if he had not fled. Since the court had often disbarred people for far less serious offenses, it had no hesitation in disbarring the attorney. In Re Hinds Facts: Attorney held a press conference and harshly criticized the trial judge's conduct of an ongoing criminal trial. Attorney had previously represented the defendant in a civil case and assisted the attorneys in the criminal trial although appellant was not attorney of record at the time of the press conference. An ethics committee recommended that appellant be charged with violations of N.J. Ct. R., R. Prof. Conduct 1.102(a)(5), and 7.107(d). Holding: The court held that record was insufficient to make factual determinations in the case and dismissed the charges, but made certain rulings to explain the appropriate ethics principles that would be applied prospectively only. The court concluded that the reasonable likelihood standard of 7.107(D) was constitutional as applied to the extrajudicial statements of an attorney who was associated with a criminal trial, where the statements were about the trial and were intended to be publicly disseminated. Reasoning: N.J. Ct. R., R. Prof. Conduct 7.107(D), restricts the speech of attorneys who are associated with pending criminal litigation by sanctioning such attorneys for making any extrajudicial statement that they expect to be disseminated to the public and that is "reasonably likely" to interfere

38

with a fair trial. There can be no doubt that the state has a substantial interest in ensuring the fairness of judicial proceedings. This interest does not belong to the defendant alone. The public also has an interest in a fair trial that cannot be imperiled or diminished by out-of-court assertions by either defense or prosecution lawyers. Thus, courts have recognized that restricting the extra-judicial statements of criminal defense attorneys relates to the government's substantial interest in preserving the proper administration of justice and the basic integrity of the judicial process. The court recognizes that extrajudicial speech by attorneys participating in a criminal case is not absolutely protected under the First Amendment, if it will have a deleterious impact upon the fairness and integrity of the proceeding. The right of the state to a fair trial cannot be imperiled or diluted by an attorney's out-of-court assertions to news media on the subject of his client's innocence. The courtroom is the place to settle the issue and comments before or during the trial which have the capacity to influence potential or actual jurors to the possible prejudice of the state are impermissible Matter of Erdmann Facts: The Appellate Division had brought a petition for charges of professional misconduct against the attorney. The Appellate Division found that language contained in a statement made by the attorney and quoted in an article published in a magazine was intemperate, vulgar and insulting and is an offense against the dignity and the integrity of the courts and tends to create disrespect for judicial officers generally thereby lessening public confidence in our legal system. Holding: The Court of Appeals of New York held that without more, isolated instances of disrespect for the law, Judges and courts expressed by vulgar and insulting words or other incivility, uttered, written, or committed outside the precincts of a court are not subject to professional discipline. However, the court also noted that perhaps persistent or general courses of conduct, even if parading as criticism, which are degrading to the law, the Bar, and the courts, and are irrelevant or grossly excessive, would present a different issue. In Re Wilkins Facts: In a footnote of a petition to the court, the attorney commented that the court of appeals opinion was so factually and legally inaccurate that one is left to wonder if the court was determined to find for the [Appellee] and said whatever was necessary to reach that conclusion. The attorney appealed and requested consideration application of the First Amendment to his offending remarks. Holding: The court declined to grant a rehearing as to the First Amendment issue, as the attorney's language did not merely argue that the court of appeals decision was factually or legally inaccurate, but ascribed bias and favoritism to the judges authoring and concurring in the majority opinion of the court of appeals, and implied that the judges manufactured a false rationale in an attempt to justify their pre-conceived desired outcome. Such aspersions transgressed the wide latitude given appellate argument, and they clearly impugned the integrity of a judge in violation of Ind. R. Prof. Conduct 8.2(a). Reasoning: Lawyers are completely free to criticize the decisions of judges. As licensed professionals, they are not free to make recklessly false claims about a judge's integrity. Spevack v. Klein Facts: Attorney refused to honor a subpoena duces tecum served on him in which he refused to produce the demanded financial records and refused to testify at the judicial inquiry. His sole defense was that the production of the records and his testimony would tend to be self-incriminating. The Appellate Division ordered the attorney disbarred on the ground that the constitutional provision against self-incrimination was not available to him. The Court of Appeals of New York affirmed. Holding: The Supreme Court held that the self-incrimination clause of the Fifth Amendment was absorbed by Fourteenth Amendment and applied to everyone. The attorney could not be disbarred if he raised his privilege against self-incrimination under the Fifth Amendment. The court also reversed the finding that alternatively disbarred petitioner because he refused to produce subpoenaed documents on the grounds that it would incriminate him. The Fifth Amendment privilege did not extend to records. However, basing disbarment on the attorney's assertion of this privilege was improper. The attorney should have been given an opportunity to show that the requested documents were outside the scope of the required records rule and were private papers with no public aspects.

39

Reasoning: The Self-Incrimination Clause of the Fifth Amendment has been absorbed in the Fourteenth Amendment, it extends its protection to lawyers as well as to other individuals, and it should not be watered down by imposing the dishonor of disbarment and the deprivation of a livelihood as a price for asserting it. Under the self-incrimination clause of the Fifth Amendment, a person has the right to remain silent without suffering any penalty for such silence; penalty in this context means the imposition of any sanction, which makes assertion of the Fifth Amendment privilege costly. Debock v. State Facts: A former assistant state attorney was subpoenaed to give testimony regarding criminal charges pending against an attorney who allegedly offered unlawful compensation to him while he was an assistant state attorney. The former assistant state attorney was granted immunity pursuant to a Florida statute. During a deposition, the attorney refused to answer certain questions and asserted his constitutional right against self-incrimination because the immunity did not protect him from any disciplinary action by the state bar. The trial court agreed and entered an order that permitted him to invoke the privilege. The appellate court reversed, stating that any disciplinary action by the state bar was remedial and not penal; therefore, appellant could not refuse to testify in a criminal proceeding. Holding: On appeal, the court affirmed the appellate court decision because the immunity granted pursuant to the Florida statute fully protected appellant from having his testimony used in any criminal proceeding against him. The court stated that if appellant refused to testify, he could be held in contempt of court. Reasoning: Membership in the bar is a privilege burdened with conditions. A fair private and professional character is one of them. Compliance with that condition is essential at the moment of admission; but it is equally essential afterwards. Whenever the condition is broken the privilege is lost. To refuse admission to an unworthy applicant is not to punish him for past offenses. The examination into character, like the examination into learning, is merely a test of fitness. To strike the unworthy lawyer from the roll is not to add to the pains and penalties of crime. The examination into character is renewed; and the test of fitness is no longer satisfied. For these reasons courts have repeatedly said that disbarment is not punishment. Bar disciplinary proceedings are remedial and are designed for the protection of the public and the integrity of the courts. An immunized attorney's testimony in a criminal proceeding can be used in a bar inquiry. Attorneys can be held to different standards than other regulated professions. The unique role of attorneys as officers of the court mandates that attorneys be held to the highest of ethical standards. An immunized attorney may be granted immunity from bar disciplinary proceedings by order of the state supreme court where it appears that the greater good to society will be served by granting immunity from disciplinary action to an attorney. It is the attorney seeking bar immunity who must so persuade the court. D. MODEL RULE 8.5 DISCIPLINARY AUTHORITY CHOICE OF LAW (a) Disciplinary Authority. A lawyer admitted to practice in this jurisdiction is subject to the disciplinary authority of this jurisdiction, regardless of where the lawyer's conduct occurs. A lawyer not admitted in this jurisdiction is also subject to the disciplinary authority of this jurisdiction if the lawyer provides or offers to provide any legal services in this jurisdiction. A lawyer may be subject to the disciplinary authority of both this jurisdiction and another jurisdiction for the same conduct. (b) Choice of Law. In any exercise of the disciplinary authority of this jurisdiction, the rules of professional conduct to be applied shall be as follows: (1) For conduct in connection with a matter pending before a tribunal, the rules of the jurisdiction in which the tribunal sits, unless the rules of the tribunal provide otherwise; (2) For any other conduct, the rules of the jurisdiction in which the lawyer's conduct occurred, or, if the predominant effect of the conduct is in a different jurisdiction, the rules of that jurisdiction shall be applied to the conduct. A lawyer shall not be subject to discipline if the lawyer's conduct conforms to the rules of a jurisdiction in which the lawyer reasonably believes the predominant effect of the lawyer's conduct will occur.

40

E. MAJOR ISSUES IN BAR DISCIPLINE Can conduct that is wholly unrelated to the practice of law purely result in bar punishment? Yes. Can invoking ones 5th Amendment right result bar punishment? Yes. Does an attorneys profession infringe on his/her First Amendment rights? Yes. Should there be an affirmative duty for an attorney to report professional misconduct? Debatable. X. Lawyers and Fundamental Moral Responsibility A. THE MORAL PERSON Consider MR 1.2(b), 2.1, 5.1 and 5.2. The Bramble Bush (Karl Llewellyn) Suggests legal education can be a narrowing experience that displaces the moral vision and perspective gained over a lifetime. The Apology of Socrates (Plato) Socrates, according to Platos account, believed that despite the difficulty of under the nature of good and evil, these qualities are facts, like the laws of physics. Just because we have difficulty in defining the nature of good does not mean that there is not difference between good and evil, or that the nature of good can be changed by democratic votes, public opinion, judicial proceedings or military victories. Once the oracle declared that no one was wiser than he, Socrates had set out to test its meaning by examining all those who claimed to be wise. Finding none who could pass the test, he concluded that the meaning of the oracle was probably that no one could be wiser than a person who was aware, as Socrates was, of his own failure to be wise. Socrates indictment was brought by three Athenian citizens: (i) Anytus, a democratic politician; (ii) Meletus, speaking on behalf of the poets; and (iii) Lycon, an orator. The official charge is that he corrupted the young men of Athens by teaching rhetortic. For Anytus, in a democracy the majority rules and democratically elected representatives enact laws based on a plurality. To say that an individual could call the acts of a duly elected majority evil and thus oppose them, attacks the system at is core. For Meletus, artists dislike absolute standards of morality. Censorship, whether by a democratic representative or a philosopher, is what they fear. For Lycon, lawyers understand the comfort of a belief that there is no good or bad side to a case, but just differing interests or even differing personal realities. This provides reassurances to all but those few advocates who never represent anyone except for those with whom they totally agree. [N]o evil can happen to a good man, either in life or after death. The Prince (Niccolo Machiavelli) For Machiavelli, there is no moral basis on which to judge the difference between legitimate and illegitimate uses of power. Rather, authority and power are essentially coequal: whoever has power has the right to command; but goodness does not ensure power and the good person has no more authority by virtue of being good. Thus, in direct opposition to a moralistic theory of politics, Machiavelli says that the only real concern of the political ruler is the acquisition and maintenance of power. Machiavelli acknowledges that good laws and good arms constitute the dual foundations of a well-ordered political system. But he immediately adds that since coercion creates legality, he will concentrate his attention on force. He says, Since there cannot be good laws without good arms, I will not consider laws but speak of arms. In other words, the legitimacy of law rests entirely upon the threat of coercive force; authority is

41

impossible for Machiavelli as a right apart from the power to enforce it. Consequently, Machiavelli is led to conclude that fear is always preferable to affection in subjects, just as violence and deception are superior to legality in effectively controlling them. Machiavelli observes that one can say this in general of men: they are ungrateful, disloyal, insincere and deceitful, timid of danger and avid of profit. Love is a bond of obligation which these miserable creatures break whenever it suits them to do so; but fear holds them fast by a dread of punishment that never passes. And therefore, he must have a mind disposed to adapt itself according to the wind, and as the variations of fortune dictate, and, as I said before, not deviate from what is good, if possible, but be able to do evil if necessitated. Invisible Man (Ralph Ellison) Ellison regards invisibility as a kind of moral sickness, the result of losing ones true identity and moral compass. [I]f you dont know where you are, you probably dont know who you are When one is invisible he finds such problems as good and evil, honest and dishonesty, of such shifting shapes that he confuses one with the other, depending on who happens to be looking through him at the time. B. MORAL RESPONSIBILITY Consider MR 1.2(b) & (d), 1.6(a) & (b), 1.8(b), 2.1, 3.1, 3.3(a), 4.1(b), 5.1(a), 5.1(c), 8.4(a) & (c). The Nicomachean Ethics (Aristotle) Aristotle assumes that human beings are fundamentally different from other animals in that they can (i) subordinate sensuality and appetite to rational rule and principle and (ii) they can use reason to seek for and contemplate truth. But it is also this human nature that demands a very high sense of responsibility. Aristotle holds all human beings responsible for any evil they have, in fact, the power to prevent. Aristotle is very strict with excuses: most coercion is not an excuse nor is ignorance. Nuremberg and Vietnam (Telford Taylor) The moral rules applied were also rules of the higher law applicable in all times and places (and thus in Germany and its territories, before as after the London Charter) as a source of argumentation and judgment according to law when the social-fact sources which are the normally dominant and quasi-exclusive source of law are, in justice, inadequate and insufficient guides to fulfilling obligations such as the judicial obligation to do justice according to law, or everyone's obligation to behave with elementary humanity even when under orders not to even if those orders have intra-systemic legal validity according to the formal or social-fact criteria of some existing legal system. And if one has doubts about victors' justice, those very doubts can likewise appeal to principles of the same higher law, ius gentium, or law of reason and humanity. Taylor describes how our overarching moral standing is jeopardized if we allow defendants to shake off culpability either by showing that what they did was not wrong, however unlawful, or that if wrong, others more highly placed were primarily responsible. C. THE LEGAL MENTALITY Legal Positivism Legal positivism is the thesis that the existence and content of law depends on social facts and not on its merits. John Austin (1790-1859) formulated it thus: The existence of law is one thing; its merit and demerit another. Whether it be or be not is one enquiry; whether it be or be not conformable to an assumed standard, is a different enquiry. The positivist thesis does not say that law's merits are unintelligible, unimportant, or peripheral to the philosophy of law. It says that they do not determine whether laws or legal systems exist.

42

Whether a society has a legal system depends on the presence of certain structures of governance, not on the extent to which it satisfies ideals of justice, democracy, or the rule of law. What laws are in force in that system depends on what social standards its officials recognize as authoritative; for example, legislative enactments, judicial decisions, or social customs. The fact that a policy would be just, wise, efficient, or prudent is never sufficient reason for thinking that it is actually the law, and the fact that it is unjust, unwise, inefficient or imprudent is never sufficient reason for doubting it. According to positivism, law is a matter of what has been posited (ordered, decided, practiced, tolerated, etc.); as we might say in a more modern idiom, positivism is the view that law is a social construction. According to Bentham, law is a phenomenon of large societies with a sovereign: a determinate person or group who have supreme and absolute de facto power they are obeyed by all or most others but do not themselves similarly obey anyone else. The laws in that society are a subset of the sovereign's commands: general orders that apply to classes of actions and people and that are backed up by threat of force or sanction. This imperatival theory is positivist, for it identifies the existence of legal systems with patterns of command and obedience that can be ascertained without considering whether the sovereign has a moral right to rule or whether his commands are meritorious. It has two other distinctive features. The theory is monistic: it represents all laws as having a single form, imposing obligations on their subjects, though not on the sovereign himself. The imperativalist acknowledges that ultimate legislative power may be self-limiting, or limited externally by what public opinion will tolerate, and also that legal systems contain provisions that are not imperatives (for example, permissions, definitions, and so on). But they regard these as part of the non-legal material that is necessary for, and part of, every legal system. The theory is also reductivist, for it maintains that the normative language used in describing and stating the law talk of authority, rights, obligations, and so on can all be analyzed without remainder in non-normative terms, ultimately as concatenations of statements about power and obedience. According to Justice Holmes, [i]f you want to know the law and nothing else, you must look at it as a bad man, who cares only for the material consequences which such knowledge enables him to predict, not as a good one, who finds his reason for conduct, whether inside the law or outside of it, in the vaguer sanctions of conscience.

The Institutes (Justinian) The law of nature (ius naturale) is the law which nature teaches to all animals. The laws of nature, which all nations observe alike, being established by a divine providence, remain ever fixed and immutable. The law of nations (ius gentium) is the law which natural reason appoints for all mankind obtains equally among all nations because all nations make use of it. The civil law (ius civile) is the law, which a people make for its own government, belonging exclusively to that state and being the law of the particular state. Knowledge and Politics (Roberto Unger) To the positivist, society has no inherent order on its own. He sees rules as the impositions of a will, even though of an enlightened one, on the chaos of social life. The universal laws are simply conventions, which set the boundaries among particular interest so that these interests will not destroy each other. The assumption of the belief that the law must be capable of coercive enforcement is the artificial view of society. According to this view, even though society may have an implicit order, as the natural rights theorist claims, it is not a self-regulating or self-enforcing one. Because individuals and individual interests are the primary elements of social life, and because they are locked in a perpetual struggle with one another, social order must be established by acts of will and protected against the ravages of self-interest.

43

The idea that there is no natural community of common ends and that group life is a creature of will, help explain the importance of rules and of their coercive enforcements. But the same factors may also account for the fascination of terror, the systematic use of violence unlimited by law, as a device of social organization. The less ones ability to rely on participation in common ends, the greater the importance of force as a bond among individuals. Punishment and fear take the place of community. The natural rights theorist, on the contrary, claims to discover an intrinsic order in social relations, an order it is his purpose to make explicit and to develop. For him the universals that describe this order rights, rules and institutional categories have an existence and worth quite independent of the particular interests that may take advantage of them. Thus, the natural rights theorist treats the system of private law concepts and property or the doctrine of separation of powers in public law as if they had an autonomous logic that survived in all their transmutations. The positivist account of legal validity is hard to reconcile with the [claim] that valid law as such, no matter what its content, deserves our respect and general fidelity. Even if valid law is bad law, we have some obligation to obey it simply because it is law. But how can this be so if a law's validity has nothing to do with its content?

The Idea of Law (Dennis Lloyd) It is useful to remember that in many less well-regulated societies the operation of law may appear in a more unfavorable guise. The Nature of Man. There are those who view mans nature as inherently good and seek to find the sources of the ills of mans present condition in situations external to man himself and hence look for some fundamental defect in mans social environment as the true cause of the evils which afflict him. The idea of vice and corruption as the reason for the establishment of coercive institutions became a key feature of Western thought for many centuries, adapted as it was by the early Church Fathers to the Judeo-Christian version of the Fall of Man. Leaning heavily upon Aristotles conception of the natural development of the state from mans social impulses, Aquinas held that the state was not a necessary evil but was a natural foundation in the development of human welfare. The Alternative Viewpoint. Marx believed that law was nothing but a coercive system devised to maintain the privileges of the property-owning class; by the revolution of a classless society would be brought into being, and law and the state would wither away as being no longer needed to support an oppressive regime. D. THE MORAL CONSTRAINTS ON RULE MAKING The Summa Theologica (St. Thomas Aquinas) St. Thomas ethical philosophy is predicated on the existence of God. God has a will, and divine law directs us to obey that will, even though people are free to be wicked and refuse to obey. Failure to recognize and obey divine law, however, is a sin. The natural law is comprised of those precepts of the eternal law that govern the behavior of beings possessing reason and free will. The first precept of the natural law, according to Aquinas, is the somewhat vacuous imperative to do good and avoid evil. Aquinas holds a natural law theory of morality: what is good and evil, according to Aquinas, is derived from the rational nature of human beings. Good and evil are thus both objective and universal. St. Thomas states that Gods will is to obey the positive law, except where to do so is to violate Gods own laws, or to reduce the law to something the law giver did not actually intend but only in clear cases.

44

A human law (i.e., that which is promulgated by human beings) is valid only insofar as its content conforms to the content of the natural law; as Aquinas puts the point: "[E]very human law has just so much of the nature of law as is derived from the law of nature. But if in any point it deflects from the law of nature, it is no longer a law but a perversion of law." This law of nature, being coeval with mankind and dictated by God himself, is of course superior in obligation to any other. It is binding over all the globe, in all countries, and at all times: no human laws are of any validity, if contrary to this; and such of them as are valid derive all their force, and all their authority, mediately or immediately, from this original.

The Concept of Law (H.L.A. Hart) Hart argued that confusing personal values and law is bad philosophy and bad jurisprudence. It cannot seriously be disputed that the development of law, at all times and places, has in fact been profoundly influenced both by the conventional morality and ideals of particular social groups, and also by forms of enlightened moral criticism urged by individuals, whose moral horizon has transcended the morality currently accepted. But, it is possible to take this truth illicitly, as a warrant for a different proposition: namely that a legal system must exhibit some specific conformity with morality or justice, or must rest on a widely diffused conviction that there is a moral obligation to obey it. But is does not follow that the criteria of legal validity of particular laws used in a legal system must include, tacitly if not explicitly, a reference to morality or justice. Hart argues for the existence of a possible legal system in which there are no moral constraints on legal validity. Hart argues that while every legal system must contain so-called primary rules that regulate citizen behavior, he believes a system consisting entirely of the kind of liberty restrictions found in the criminal law is, at best, a rudimentary or primitive legal system. As Hart points out, the rules governing the creation of contracts and wills cannot plausibly be characterized as restrictions on freedom that are backed by the threat of a sanction. These rules empower persons to structure their legal relations within the coercive framework of the law-a feature that Hart correctly regards as one of "law's greatest contributions to social life." Legal rules are obligatory, according to Hart, because people accept them as standards that justify criticism and, in extreme cases, punishment of deviations: What is necessary is that there should be a critical reflective attitude to certain patterns of behavior as a common standard, and that this should display itself in criticism (including self-criticism), demands for conformity, and in acknowledgements that such criticism and demands are justified, all of which find their characteristic expression in the normative terminology of 'ought', 'must', and 'should', and 'right' and 'wrong'. Hart argues that what is necessary to the existence of a legal system is that the majority of officials take the internal point of view towards the rule of recognition and its criteria of validity. All that is required of citizens is that they generally obey the primary rules that are legally valid according to the rule of recognition. Thus, on Hart's view, there are two minimum conditions sufficient and necessary for the existence of a legal system: "On the one hand those rules of behavior which are valid according to the system's ultimate criteria of validity must be generally obeyed, and, on the other hand, its rules of recognition specifying the criteria of legal validity and its rules of change and adjudication must be effectively accepted as common public standards of official behavior by its officials." Hart rejects Austin's view because the institutional application of coercive force can no more give rise to an obligation than can the application of coercive force by a gunman. But the situation is no different if the gunman takes the internal point of view towards his authority to make such a threat. Despite the gunman's belief that he is entitled to make the threat, the victim is obliged, but not

45

obligated, to comply with the gunman's orders. The gunman's behavior is no less coercive because he believes he is entitled to make the threat. And likewise for a minimal legal system where only the officials of the legal system take the internal point of view towards the rule of recognition that endows them with authority to make, execute, adjudicate, and enforce the rules. The mere presence of a belief in the officials that they are entitled to make law cannot give rise to an obligation in other people to comply with their enactments any more than the presence of a belief on the part of a gunman that he is entitled to issue orders gives rise to an obligation in the victim to comply with those orders. Hart's minimal legal system is no less coercive than Austin's legal system.

The Morality of Law (Lon Fuller) Fuller articulates the eight pitfalls of rulemaking, which include: (i) a failure to achieve rules at all, so that every issue must be decided on an ad hoc basis; (ii) a failure to publicize, or at least make available to the affected party, the rules he is expected to observe; (iii) the abuse of retroactive legislation, which cannot provide a guide for future action but undercuts the integrity of rules prospective effect, since it puts them under threat of retrospective change; (iv) a failure to make rules understandable; (v) the enactment of contradictory rules or (vi) rules that require conduct beyond the powers of the affected party; (vii) introducing such frequent changes in the rules that the subject cannot orient his action by them; and, finally, (viii) a failure of congruence between the rules announced and their actual administration. A total failure in any one of these eight directions does not simply result in a bad system of law; it results in something that is not properly called a legal system at all" Lon Fuller rejects the conceptual naturalist idea that there are necessary substantive moral constraints on the content of law. But Fuller believes that law is necessarily subject to a procedural morality. On Fuller's view, human activity is necessarily goal-oriented or purposive in the sense that people engage in a particular activity because it helps them to achieve some end. Insofar as human activity is essentially purposive, according to Fuller, particular human activities can be understood only in terms that make reference to their purposes and ends. Thus, since lawmaking is essentially purposive activity, it can be understood only in terms that explicitly acknowledge its essential values and purposes. To the extent that a definition of law can be given, then, it must include the idea that law's essential function is to "achiev[e] social order through subjecting people's conduct to the guidance of general rules by which they may themselves orient their behavior." These internal principles constitute a morality, according to Fuller, because law necessarily has positive moral value in two respects: (1) law conduces to a state of social order and (2) does so by respecting human autonomy because rules guide behavior. Since no system of rules can achieve these morally valuable objectives without minimally complying with the principles of legality, it follows, on Fuller's view, that they constitute a morality. Since these moral principles are built into the existence conditions for law, they are internal and hence represent a conceptual connection between law and morality. E. NATURAL LAW: THE LAW IN THE SKY AND IN THE CAVE Of the Rights of War and Peace (Hugo Grotius) Grotius set out to demonstrate that only a theory of natural law, which is independent of the positivist test of sovereignty and coercion, would suffice in making international law valid law.

46

Grotius also set out to demonstrate a natural law based on mans own nature and independent of God, and that on the basis of the law of nature it was possible to formulate a coherent code suitable for all times and places. For Grotius, a study of nature itself and more specifically, a study of human nature can suffice to teach us the essentials of ethics, politics and law. This law would not only bind all nations, but would provide a test as to the rightness of all positive legal systems. Grotius also argued that this natural law could be discovered by examining that which is generally believed to be so by all, or at least, the most civilized Nations. The power of the collective human mind and of its consequence, civilization, can establish the difference between intrinsically good and evil. The law of nature is a dictate of right reason, which points out that an act, according as it is or is not in conformity with rational nature, has in it a quality of moral baseness or moral necessity; and that, in consequence, such an act is either forbidden or enjoined." If an action agrees with the rational and social aspects of human nature, it is permissible; if it doesn't, it is impermissible. That is to say, the source of the natural law is the in/compatibility of actions with our essences as rational and social beings. Human nature is constituted by two essential properties: the desire for selfpreservation and the need for society. These two properties temper and inform each other: the desire for self-preservation is limited by the social impulse, so that humans do not naturally seek to maintain and enhance their being at all costs; conversely, the need for the company of other humans is limited by the self-preservation drive, for individuals must naturally strive to secure the means for their well-being. Because we are essentially both social and self-preserving beings, it follows that two things are imperative for our successful existence. We ought to abstain from what belongs to other persons, and we ought to engage in the reasonable pursuit of what genuinely serves our interest. Accordingly, Grotius makes these the first two of four elements of natural law. Grotius argued that unlike natural law theorist who tended to invoke God or positivists who invoked the nature of a superior/inferior relationship individuals are obliged to perform or avoid certain actions, which are compatible or incompatible with our natures as rational beings, because we will be less than human if we don't. Grotius adopts this idea for himself, modifying it in keeping with his rich conception of human nature: for him, natural law obliges us to perform actions which conduce to our rationality, sociability and need for self-preservation. This thought underlies several of his specific laws of nature such as, for instance, the law that evil deeds must be corrected. For him, natural laws apply to all rational and social beings as such. It doesn't matter what they think or believe; if they are rational and social, they are bound by the law of nature. The content of the law of nature is so unalterable, that God himself cannot change it For instance then, as twice two should not be four, God himself cannot effect; so neither can he, that what is intrinsically evil, should not be evil. N.B. Justice Holmes said the jurists who believe in natural law seem to me to be in that nave state of mind that accepts what has been familiar and accepted by them and their neighbors as something that must be accepted by all men everywhere.

Natural Law (A.P. dEntrves) A natural law moral theory will give an account of the way in which first principles of practical reason take on a moral force by being considered, not one by one but in their united (integral) directiveness. That integral directiveness is given specific (albeit highly general) articulation in principles such as the injunction to love one's neighbor as oneself; or the Golden Rule of doing for others what you would want them to do for you and not doing to others what you would not have them do to you; or the categorical imperatives to respect, and treat as intrinsically valuable, humanity (the basic aspects of human flourishing) in oneself and in others, so that each of one's communities is treated as a kingdom of ends of persons each ends in themselves.

47

Natural Law (Oliver Wendell Holmes, Jr.) [T]he truth may be defined as the system of my (intellectual) limitations, what give it objectivity is the fact that I find my fellow man to a greater or less extent (never wholly) subject to the same cant helps. Deep-seated preferences can not be argued about you can not argue a man into liking a glass of beer and therefore, when we try to kill the other man rather than let him have his way. But, that is perfectly consistent with admitting that, so far as appears, his grounds are just as good as ours. I see no a priori duty to live with others and in that way [society], but simply a statement of what I must do if I wish to remain alive. If I do live with others they tell me that I must do and abstain from doing various things or they will put to the screws to me. I believe that they will, and being of the same mind as to their conduct I not only accept the rules but come in time to accept them with sympathy and emotional affirmation and begin to talk about rights and duties. No doubt behind these rights is the fighting will of the subject to maintain them, and the spread of his emotions to the general rules by which they are maintained; but that does not seem to me the same thing as the supposed a priori discernment of a duty or the assertion of a preexisting right. A dog will fight for his bone. I do not see any rational ground for demanding the superlative for being dissatisfied unless we are assured that our truth is cosmic truth, if there is such a thing that the ultimates of a little creature on this little earth are the last word of the unimaginable whole. The Case of the Speluncean Explorers (Lon Fuller) Chief Justice Truepenny Advocate of Textualism/Institutionalism. [Affirm] Chief Justice Truepenny votes to uphold the verdict because the statute permits of no exception applicable to this case, how-ever our sympathies may incline us to make allowance for the tragic situation in which these men found themselves. But the Chief Justice also urges the Court to join in the clemency petition to the Chief Executive. In this way, justice will be accomplished without impairing either the letter or spirit of our statutes and without offering any encouragement for the disregard of law. Justice Keen Advocate of Positivism and/or Textualism. [Affirm] Justice Keen provides the second vote to affirm the convictions. At the outset of his opinion, Justice Keen quickly disposes of two issues that he says are not the province of the Court: (1) executive clemency [although Justice Keen rejects the call for executive clemency, he notes that if he were the Chief Executive rather than a Supreme Court Justice, he would go further than the six-months imprisonment sought in the clemency petition and instead pardon these men altogether, since I believe that they have already suffered enough to pay for any offense they may have committed] and (2) moralitywhether what these men did was right or wrong. Instead, the sole issue is one of statutory construction, and Justice Keen concludes that any candid observer, content to extract from these words their natural meaning, would concede at once that these defendants did willfully take the life of Roger Whetmore. Justice Keen chastises his colleagues for their failure to distinguish the legal from the moral aspects of this case. He bluntly accuses his fellow Justices of subordinating the law of the land to their own conceptions of morality. Justice Keen recounts the civil war fought in Newgarth centuries earlier as a result of an unseemly struggle for power between the judicial branch and the legislative and executive branches, which established the supremacy of the legislature. He, like Justice Tatting, sharply criticizes Justice Foster for disregarding the clear language of the statute in order to further its purported purpose of deterrence. According to Justice Keen, the statute at issue here, like most if not all statutes, has multiple purposes. It is thus, a canard to argue that

48

courts may fill in the gap in the statute or make corrections in the legislative design. Instead, the Courts role is to interpret the statute in accordance with its plain meaning without reference to our personal desires or our individual conceptions of justice. Justice Keen also takes issue with Justice Fosters and Justice Tattings reliance on an exception to the murder statute, created by an earlier decision of the Court on the theory that a defendant who acts in self-defense does not act willfully. Justice Keen contends that the exception is not relevant here because it applies only when the defendant resists a threat to his own life, and Whetmore made no threat against the lives of these defendants. Justice Keen concludes that his approach provides the best result in this case and leads to the sounder ad-ministration of justice in the long run. [W]e would have inherited a better legal system from our forefathers if [these] principles had been observed from the beginning. For example, with respect to the excuse of self-defense, if our courts had stood steadfast on the language of the statute the result would undoubtedly have been a legislative revision of it. Justice Foster Advocate of Natural Law/Purposive Analysis. [Reverse] Justice Foster first criticizes Chief Justice Truepennys exhortation to seek clemency as an expedient at once so sordid and so obvious:
I believe something more is on trial in this case than the fate of these unfortunate explorers; that is the law of our Commonwealth. If this Court declares that under our law these men have committed a crime, then our law is itself convicted in the tribunal of common senseFor us to assert that the law we uphold and expound compels us to a conclusion we are ashamed of, and from which we can only escape by appealing to a dispensation resting within the personal whim of the Executive, seems to me to amount to an admission that the law of this Commonwealth no longer pretends to incorporate justice.

Justice Foster then offers two independent justifications in support of his view that the law does not compel the monstrous conclusion that these men are murderers. First, the statute and related case law do not apply because the explorers horrendous circumstances placed them in a state of nature, and the explorers thus were subject only to natural law. Under the natural law precept of freedom of contract, the explorers compact was justifiable to enable the four to survive at the cost of the one. Second, the statute must be interpreted in light of its purpose of deterring murder, and this purpose is not served in convicting the defendants because they were justified in taking Whetmores life to ensure their own survival. Although Justice Foster professes fealty to the principle that the Court is bound by statutes and is thus subservient to the duly expressed will of the legislature, he distinguishes intelligent from unintelligent obedience. Justice Foster emphasizes that [n]o superior wants a servant who lacks the capacity to read between the lines. Justice Foster offers two examples: The stupidest housemaid knows that when she is told to peel the soup and skim the potatoes her mistress does not mean what she says. She also knows that when her master tells her to drop every-thing and come running he has overlooked the possibility that she is at the moment in the act of rescuing the baby from the rain barrel. Surely we have a right to expect the same modicum of intelligence from the judiciary. He thus concludes that the correction of obvious legislative errors or oversights is not to supplant the legislative will, but to make that will effective [A man may break the letter of the law without breaking the law itself]. Justice Handy Advocate of Practical Wisdom/Common Sense. [Reverse] Justice Handy takes a pragmatic, common-sense approach in providing the second vote to reverse the convictions. Justice Handy notes that he has become more and more perplexed at mens refusal to apply their common sense to problems of law and government. The question on appeal, he writes, is one of practical wisdom, to be exercised in a context, not of abstract theory, but of realities. He chides his colleagues for throwing an obscuring curtain of

49

legalisms at what is at bottom a simple case. For example, he criticizes the learned disquisitions on the distinction between positive law and the law of nature, the language of the statute and the purpose of the statute, judicial functions and executive functions, judicial legislation and legislative legislation. Justice Handy reminds his colleagues of the danger of getting lost in the patterns of our thought and forget that these patterns often cast not the slightest shadow on the outside world. In Justice Handys view, courts derive their legitimacy by bending to popular will, and the Court here should follow the poll reporting that 90% of the public believes that the defendants should be pardoned or given a token punishment. Justice Handy latches onto Justice Fosters statutory purpose argument to provide the legal rationale for this result. Justice Handy concludes his opinion with two lessons drawn from his personal experience. First, he dismisses the possibility of clemency based on gossip from his wifes niece, who is a friend of the Chief Executives secretary. According to Justice Handy, he shared this information with his colleagues. To him this explained Chief Justice Truepennys flap[ping] his judicial robes to encourage the Chief Executive to grant clemency, [Justice] Fosters feat of levitation by which a whole library of law books was lifted from the shoulders of these defendants, and [Justice] Keen emulat[ing] Pooh-Bah in the ancient comedy by stepping to the other side of the stage to address a few remarks to the Executive in my capacity as a private citizen. Second, he compares this case to the first case he heard as a trial judge, in which he was widely approved by the press and public opinion for employing his common sense and avoiding the many perplexing legal issues raised in the case. Justice Tatting Abstaining from Decision. [No Vote] With the Court deadlocked, the deciding vote falls to Justice Tatting. His initial opinion is written after the first opinions to affirm (Chief Justice Truepenny) and reverse (Justice Foster). He finds himself torn between sympathy for [the defendants] and a feeling of abhorrence and disgust at the monstrous act they committed. He had hoped to be able to put these contradictory emotions to one side as irrelevant, and to decide the case on the basis of a convincing and logical demonstration of the result demanded by our law. But he is not convinced by the arguments on either side. Justice Tatting rejects both of Justice Fosters rationales for reversing the convictions. The question of the boundary of a state of nature is intractable, and in any event Justice Tatting is not persuaded that the law applicable in such a state should permit the law of contracts to override the law of murder. The purposive analysis is unavailing where, as here, there are several purposes served by the criminal statute (retribution and rehabilitation in addition to deterrence). Moreover, the self-defense rationale is flawed because the defendants here acted willfully and deliberately in planning and executing Whetmores killing. Justice Tatting invokes the case of Jean Claude Valjean of Les Misrables in arguing that impending starvation neither excuses Valjeans stealing of a loaf of bread nor the defendants killing of Whetmore. Justice Tatting envisions a quagmire of hidden difficulties if the self-defense exception is applied. Justice Tatting ultimately rejects Justice Fosters arguments in favor of reversal as intellectually unsound and approaching mere rationalization. Yet Justice Tatting also is repelled at the prospect of affirming the convictions. He complains: [T]he more I examine this case and think about it,[m]y mind becomes entangled in the meshes of the very nets I throw out for my own rescue. I find that almost every consideration that bears on the decision of the case is counterbalanced by an opposing consideration leading in the opposite direction. After expressing regret that the prosecutor did not simply refuse to indict the defendants, Justice Tatting takes the unprecedented step of withdrawing from the case. After the second opinions to affirm (Justice Keen) and re-verse (Justice Handy) are proffered, Chief Justice Truepenny asks Justice Tatting to reconsider. Justice

50

Tatting declines: [A]fter hearing these opinions I am greatly strengthened in my conviction that I ought not to participate in the decision of this case. As a result, the Court is evenly divided, the convictions are affirmed, and the executions are set for April 2, 4300. F. JUSTIFIED DISOBEDIENCE Consider MR 8.3(a) and 8.4. The Works of John Locke (John Locke) Locke believed that men hand over to a legislative and executive power the authority to pass and to enforce laws, which will protect their natural rights. But Lockes social contract is conditional, for insofar as the civil authority does not protect natural rights, it ceases to be a legitimate authority. The state of nature knows no government; but in it, as in political society, men are subject to the moral law, which is the law of God. Men are born free and equal in rights. Locke sees that, when men have multiplied and land has become scarce, rules are needed beyond those which the moral law or law of nature supplies. But the origin of government is traced not to this economic necessity, but to another cause. The moral law is always valid, but it is not always kept. In the state of nature all men equally have the right to punish transgressors: civil society originates when, for the better administration of the law, men agree to delegate this function to certain officers. Thus government is instituted by a "social contract"; its powers are limited, and they involve reciprocal obligations; moreover, they can be modified or rescinded by the authority, which conferred them. From this natural state of freedom and independence, Locke stresses individual consent as the mechanism by which political societies are created and individuals join those societies. While there are of course some general obligations and rights that all people have from the law of nature, special obligations come about only when we voluntarily undertake them. Locke suggests that we voluntarily undertake them through our tacit consent. Leviathan (Thomas Hobbes) According to Hobbes, the justification for political obligation is this: given that men are naturally self-interested, yet they are rational, they will choose to submit to the authority of a Sovereign in order to be able to live in a civil society, which is conducive to their own interests. Hobbes argues for this by imagining men in their natural state, or in other words, the State of Nature. In the State of Nature, which is purely hypothetical according to Hobbes, men are naturally and exclusively self-interested, they are more or less equal to one another, (even the strongest man can be killed in his sleep), there are limited resources, and yet there is no power able to force men to cooperate. Given these conditions in the State of Nature, Hobbes concludes that the State of Nature would be unbearably brutal. In the State of Nature, every person is always in fear of losing his life to another. They have no capacity to ensure the long-term satisfaction of their needs or desires. No long-term or complex cooperation is possible because the State of Nature can be aptly described as a state of utter distrust. Given Hobbes' reasonable assumption that most people want first and foremost to avoid their own deaths, he concludes that the State of Nature is the worst possible situation in which men can find themselves. It is the state of perpetual and unavoidable war. The situation is not, however, hopeless. Because men are reasonable, they can see their way out of such a state by recognizing the laws of nature, which show them the means by which to escape the State of Nature and create a civil society. The first and most important law of nature commands that each man be willing to pursue peace when others are willing to do the same, all the while retaining the right to continue to pursue war when others do not pursue peace. Being reasonable, and recognizing the rationality of this basic precept of reason, men can be expected to construct a Social Contract that

51

will afford them a life other than that available to them in the State of Nature. This contract is constituted by two distinguishable contracts. First, they must agree to establish society by collectively and reciprocally renouncing the rights they had against one another in the State of Nature. Second, they must imbue some one person or assembly of persons with the authority and power to enforce the initial contract. In other words, to ensure their escape from the State of Nature, they must both agree to live together under common laws, and create an enforcement mechanism for the social contract and the laws that constitute it. Since the sovereign is invested with the authority and power to mete out punishments for breaches of the contract which are worse than not being able to act as one pleases, men have good, albeit selfinterested, reason to adjust themselves to the artifice of morality in general, and justice in particular. Society becomes possible because, whereas in the State of Nature there was no power able to "overawe them all", now there is an artificially and conventionally superior and more powerful person who can force men to cooperate. While living under the authority of a Sovereign can be harsh (Hobbes argues that because men's passions can be expected to overwhelm their reason, the Sovereign must have absolute authority in order for the contract to be successful) it is at least better than living in the State of Nature. And, no matter how much we may object to how poorly a Sovereign manages the affairs of the state and regulates our own lives, we are never justified in resisting his power because it is the only thing which stands between us and what we most want to avoid, the State of Nature.

Civil Disobedience (Henry David Thoreau) Thoreau observes that only a very few people heroes, martyrs, patriots, reformers in the best sense serve their society with their consciences, and so necessarily resist society for the most part, and are commonly treated by it as enemies. Although at times it sounds as if Thoreau is advocating anarchy, what he demands is a better government, and what he refuses to acknowledge is the authority of one that has become so morally corrupt as to lose the consent of those governed. There will never be a really free and enlightened State, he argues, until the State comes to recognize the individual as a higher and independent power, from which all its own power and authority are derived, and treats him accordingly. There are simply more sacred laws to obey than the laws of society, and a just government should there ever be such a thing would not be in conflict with the individual conscience. Political institutions are regarded by Thoreau with distrust, and although he probably overestimates the extent to which it is possible to disassociate oneself from them, he convincingly insists that social consensus is not a guarantee of rectitude or truth. Passively allowing an unjust practice to go on is tantamount to collaborating with evil. The lawyers truth is not truth, but consistency, or a consistent expediency. Truth is always in harmony with herself, and is not concerned chiefly to reveal the justice that may consist with wrong-doing. The authority of government, even such as I am willing to submit to for I will cheerfully obey those who know and can do better than I, and in many things even those who neither know nor can do so well is still an impure one: to be strictly just, it must have the sanction and the consent of the governed. It can have no pure right over my person and property but what I concede to it. I please myself with imagining a State at last which can afford to be just to all men, and to great the individual with respect as a neighbor; which even would not think it inconsistent with its own repose, if a few were to live aloof from it, and not meddling with it, nor embraced by it, who fulfilled all the duties of neighbors and fellowmen.

52

Letter from Birmingham Jail (Martin Luther King, Jr.) As Martin Luther King, Jr. observed: If you confront a man who has been cruelly misusing you, and say Punish me, if you will; I do not deserve it, but I will accept it, so that the world will know I am right and you are wrong, then you wield a powerful and just weapon. One who breaks an unjust law must do so openly, lovingly, and with a willingness to accept the penalty. I submit that an individual who breaks a law that conscience tells him is unjust and who willingly accepts the penalty of imprisonment in order to arouse the conscience of the community over its injustice is in reality expressing the highest respect for law. "Nonviolent direct action seeks to create such a crisis and foster such a tension that a community which has constantly refused to negotiate is forced to confront the issue. It seeks to so dramatize the issue that it can no longer be ignored." One has not only a legal but a moral responsibility to obey just laws. Conversely, one has a moral responsibility to disobey unjust laws. I would agree with St. Augustine that "an unjust law is no law at all" Injustice anywhere is a threat to justice everywhere. Just as Socrates felt it was necessary to create a tension in the mind so that individuals could shake off the bondage of myths and half-truths and rise to the realm of creative analysis and objective appraisal, so must we see the need for nonviolent gadflies to create the kind of tension that will help men rise from the dark depths of prejudice and racism to the majestic heights of understanding and brotherhood. Justice too long delayed is justice denied. A just law is man-made code that squares with the moral law or the law of God. An unjust law is a code that is out of harmony with the moral law. To put it in the terms of St. Thomas Aquinas, an unjust law is a human law that is not rooted in eternal and natural law. Any law that uplifts human personality is just. Any law that degrades human personality is unjust. Taking Rights Seriously (Ronald Dworkin) Dworkin rests the right to civil disobedience not just on a person's right to political participation, but on all of the rights that he has against his government. People may be supposed to have a fundamental right against the government, such as freedom of expression, when that right is important to their dignity, to their standing as persons equally entitled to concern and respect, or to some other personal value of consequence. A person has a right to disobey a law, says Dworkin, whenever that law wrongly invades his rights against the government. Thus, the moral right to breach the law is not a separate right, like a right of conscience, additional to other rights against the government. It is that part of people's rights against the government which the government fails to honor. Dworkin wrote that, 'rights are best understood as trumps over some background justification for political decisions that states a goal for the community as a whole.' Dworkin argues considerations of rights claims must take priority over alternative considerations when formulating public policy and distributing public benefits. Thus, for example, a minority's possession of rights against discriminatory treatment should trump any and all considerations of the possible benefits that the majority would derive from discriminating against the minority group. Similarly, an individual's right to an adequate diet should trump other individuals desires to eat lavish meals, despite the aggregate gain in pleasure these individuals would derive. For Dworkin, rights as trumps expresses the fundamental ideal of equality upon which the contemporary doctrine of human rights rests. Treating rights as trumps is a means for ensuring that all individuals are treated in an equal and like fashion in respect of the provision of fundamental human rights. Fully realizing the aspirations of human rights may not require the provision of 'state of the art' resources, but this should not

53

detract from the force of human rights as taking priority over alternative social and political considerations. Dworkins main idea is that the very distinction between facts and values in the legal domain, between what the law is and what it ought to be, is much more blurred than Legal Positivism would have it: Determining what the law is in particular cases inevitably depends on moral-political considerations about what it ought to be. Evaluative judgments partly determine what the law is. Legal positivism envisaged, Dworkin claimed, that the law consists of rules only. However, this is a serious mistake, since in addition to rules, law is partly determined by legal principles. The distinction between rules and principles is basically a logical one. Rules, Dworkin maintained, apply in an all or nothing fashion. If the rule applies to the circumstances, it determines a particular legal outcome. If it does not apply, it is simply irrelevant to the outcome. On the other hand, principles do not determine an outcome even if they clearly apply to the pertinent circumstances. Principles basically provide the judges with a reason to decide the case one way or the other, and hence they only have a dimension of weight. That is, the reasons provided by the principle may be relatively strong, or weak, but they are never absolute. Such reasons, by themselves, cannot determine an outcome, as rules do. According to Dworkin's theory, unlike legal rules, which may or may not have something to do with morality, principles are essentially moral in their content. It is, in fact, partly a moral consideration which determines whether a legal principle exists or not. Why is that? Because a legal principle exists, according to Dworkin, if the principle follows from the best moral and political interpretation of past judicial and legislative decisions in the relevant domain. In other words, legal principles occupy an intermediary space between legal rules and moral principles. Legal rules are posited by recognized institutions and their validity derives from their enacted source. Moral principles are what they are due to their content, and their validity is purely content dependent. Legal principles, on the other hand, gain their validity from a combination of source-based and content-based considerations. As Dworkin put it in the most general terms: According to law as integrity, propositions of law are true if they figure in or follow from the principles of justice, fairness, and procedural due process that provide the best constructive interpretation of the community's legal practice. The validity of a legal principle then, derives, from a combination of facts and moral considerations. The facts concern the past legal decisions which have taken place in the relevant domain, and the considerations of morals and politics concern the ways in which those past decisions can best be accounted for by the correct moral principles. Needless to say, if such an account of legal principles is correct, the separation thesis can no longer be maintained. But many legal philosophers doubt that there are legal principles of the kind Dworkin envisaged. There is an alternative, more natural way to account for the distinction between rules and principles in the law: the relevant difference concerns the level of generality, or vagueness, of the norm-act prescribed by the pertinent legal norm. Legal norms can be more or less general, or vague, in their definition of the norm-act prescribed by the rule, and the more general or vague they are, the more they tend to have those quasi-logical features Dworkin attributes to principles. In the 1980's Dworkin radicalized his views about these issues, striving to ground his anti-positivist legal theory on a general theory of interpretation, and emphasizing law's profound interpretative nature. The main argument consists of two main premises. The first thesis maintains that determining what the law requires in each and every particular case necessarily involves an interpretative reasoning. Any statement of the form According to the law in S, x has a right/duty etc., to y is a conclusion of some interpretation or other. Now, according to the second premise, interpretation always involves evaluative considerations. More precisely, perhaps, interpretation is neither purely a matter of determining facts, nor is it a matter of evaluative judgment per se, but an inseparable mixture of both. Clearly enough, one who accepts both these theses, must conclude that the separation thesis is fundamentally flawed. If Dworkin is correct

54

about both theses, it surely follows that determining what the law requires always involves evaluative considerations. Dworkin maintains that the dependence of legal validity on moral considerations is an essential feature of law, which basically derives from law's profoundly interpretative nature. It may be worth noting that those legal theories maintaining that legal validity partly depends on moral considerations must also share a certain view about the nature of morality. Namely, they must hold an objective stance with respect to the nature of moral values. Otherwise, if moral values are not objective and legality depends on morality, legality would also be rendered subjective, posing serious problems for the question of how to identify what the law is. Some legal theories, however, do insist on the subjectivity of moral judgments, thus embracing the skeptical conclusions that follow about the nature of law. According to these skeptical theories, law is, indeed, profoundly dependent on morality, but, as these theorists assume that morality is entirely subjective, it only demonstrates how the law is also profoundly subjective, always up for grabs, so to speak.

The Hard Cases (Ronald Dworkin) Dworkin wants to mount a general attack on positivism organized around the observation that when lawyers reason or dispute about legal rights and obligations, particularly in those hard cases when our problems with these concepts seem most acute, they make use of standards that do not function as rules, but operate differently as principles, policies, and other sorts of standards. Dworkin believes that since positivism explains legality exclusively in terms of official acts, it cannot account for the legality of such standards. Principles v. Policies. Dworkin distinguishes principles and policies in the following way: I call a policy that kind of standard that sets out a goal to be reached, generally an improvement in some economic, political or social feature of the community. I call a principle a standard that is to be observed, not because it will advance or secure an economic, political, or social situation deemed desirable, but because it is a requirement of justice or fairness or some other dimension of morality. Thus, for example, the firm should promote job stability is a policy, while no person shall profit from her own wrong is a principle. Principles v. Rules. The distinction between rules and principles has to do with the way in which such standards are applied in deciding a case. Rules are applicable in an all-or-nothing way; if the facts stipulated by the rule are given, the rule supplies a conclusive answer. Civil Disobedience and the Lawyers Obligation to the Law (Judith McMorrow) If indeed lawyers have two special obligations to uphold the law and to work to ensure the law is just then we understandably are confused about whether lawyers should engage in civil disobedience or should counsel clients to engage in civil disobedience. [A] lawyer does not avoid the problem of special responsibility simply by asserting publicly that the act of civil disobedience is being done as a citizen, not a lawyer. Given that law and morality do not always dictate the same behavior and given the possible existence of unjust laws within a valid legal system, individual lawyers must define some standards for themselves. Requiring a lawyer to raise the possibility of civil disobedience would be like compelling a doctor who personally is opposed to abortion to tell a patient about the possibility of an illegal late-term abortion [Coquillette disagrees].

55

G. THE VALUE OF ALLEGIANCE Consider MR 1.2, 2.1, 3.1, 3.2, 3.3, 8.3 and 8.4. Crito (Plato) Socrates makes a compelling argument as to why he must stay in prison and accept the death penalty, rather than escape and go into exile in another Greek city. He personifies the Laws of Athens, and, speaking in their voice, explains that he has acquired an overwhelming obligation to obey the Laws because they have made his entire way of life, and even the fact of his very existence, possible. Socrates' life and the way in which that life has flourished in Athens are each dependent upon the Laws. Importantly, however, this relationship between citizens and the Laws of the city are not coerced. Citizens, once they have grown up, and have seen how the city conducts itself, can choose whether to leave, taking their property with them, or stay. Staying implies an agreement to abide by the Laws and accept the punishments that they mete out. And, having made an agreement that is itself just, Socrates asserts that he must keep to this agreement that he has made and obey the Laws, in this case, by staying and accepting the death penalty. Importantly, the contract described by Socrates is an implicit one: it is implied by his choice to stay in Athens, even though he is free to leave. Now you depart in innocence, a sufferer and not a doer of evil; a victim, not of the laws, but of men. But if you go forth, returning evil for evil, and injury for injury, breaking the covenants and agreements which have made with us, and wronging those whom you ought least to wrong, that is to say, yourself, your friends, your country, and us, we shall be angry with you while you live, and our brethren, the laws in the world below, will receive you as an enemy; for they will know that you have done your best to destroy us. H. DUTY FOR DUTYS SAKE Consider MR Preamble and Scope. On the Old Saw: Right in Theory But It Wont Work in Practice (Immanuel Kant) Kant makes explicit that the supreme moral principle itself must be discovered a priori, through a method of pure moral philosophy. Kant has in mind a philosophy grounded exclusively on principles that are inherent in and revealed through the operations of reason. According to Kant, the will of a moral agent is autonomous in that it both gives itself the moral law (it is self-legislating) and can constrain or motivate itself to follow the law (it is self-constraining or self-motivating). Kant believes that morality presents itself to human agents as a categorical imperative, and that it is from this imperative, together with various facts about the world and our embodied agency, that we derive all specific moral duties. Kant says that the supreme moral principle is, for rational beings who do not necessarily follow the moral law, a categorical imperative (CI). It is an imperative because it commands and constrains us; it is a categorical imperative because it commands and constrains us absolutely, with ultimate authority and without regard to our preferences or empirical features or circumstances. "Act only according to that maxim by which you can at the same time will that it should become a universal law." What we must do in any situation of moral choice is act according to a maxim that we would will everyone to act according to. This is the principle which motivates a good will, and which Kant holds to be the fundamental principle of all of morality. "Act as though the maxim of your action were by your will to become a universal law of nature." A guide for us in moral matters is to think of what would not be possible to will universally. Maxims that fail the test of the categorical imperative generate a contradiction. Laws of nature cannot be contradictory. If a maxim cannot be willed to be a law of nature, it is not moral. Act that you use humanity, whether in your own person or in the person of any other, always at the same time as an end, never merely as a

56

means. Insofar as they possess a rational will, people are set off in the natural order of things. They are not merely subject to the forces that act upon them; they are not merely means to ends. They are ends in themselves. All means to an end have a merely conditional worth because they are valuable only for achieving something else. The possessor of a rational will, however, is the only thing with unconditional worth. The possession of rationality puts all beings on the same footing, "every other rational being thinks of his existence by means of the same rational ground which holds also for myself; thus it is at the same time an objective principle from which, as a supreme practical ground, it must be possible to derive all laws of the will." A hypothetical imperative, by contrast, expresses a command of reason, but only in relation to an end already set by the agent, e.g., based on his inclinations (i.e., A hypothetical imperative says that if you wish to buy a new car, then you must determine what sort of cars are available for purchase. But Kant argues that our actions cannot be moral on the ground of some conditional purpose or goal. Morality requires an unconditional statement of one's duty.).

My Station and Its Duties (F.H. Bradley) In a Hegelian account of the moral life, Bradley argues that the self is fully realized by fulfilling its role in the social organism which grounds its duties. We must say that a mans life with its moral duties is in the main filled up by his station in that system of wholes which the state is, and that this, partly by its laws and institutions, and still more by its spirits, gives him the life which he does live and ought to live. The Original Position and Social Contract Doctrine (John Rawls) Rawls tries to show men and women, completely ignorant of his or her own characteristics and personality, acting rationally and only in his or her self-interest would choose two principles of justice. Every person must have the largest political liberty compatible with a like liberty for all; and that Inequalities in power, wealth, income and other resources must not exist except insofar as they work to the absolute benefit of the worst off members of society. Rawls says that justice as fairness assigns a certain primacy to the social. Rawls's idea is that, being reasonable and rational, persons who regard themselves as free and equal should be in a position to accept as morally justifiable the principles of justice regulating our basic social institutions and individual conduct. Otherwise their conduct is coerced for reasons they cannot (reasonably or rationally) accept and they are not fundamentally free persons.

57

You might also like